You are on page 1of 154

LECTURE 1

IMPROPER INTEGRALS AND THE MEAN VALUE THEOREM OF INTEGRAL


CALCULUS

1.0 Introduction

Dear learner, once again we welcome you to a series of lecture notes in advanced calculus
course. This forms our first lecture unit in this course which is a continuation of lecture notes in
Calculus Courses we did in our first year. We would wish to mention that it is paramount that
you understand basic concepts of Calculus I, II and III before attempting this course. In this
lecture you are introduced to improper integrals and their applications covered in detail. Several
elaborate examples are given for illustration and sufficient exercises together with answers are
also given.

Objectives

At the end of this lecture you should be able to:


1 Define an improper integral
2 Solve problems concerning improper integrals with infinite limits of integration and
infinite discontinuities.
3 Apply improper integrals to real life situations and
4 State and apply the mean value theorem of integral calculus.

1.1. Definition and some examples of Improper Integrals

b
So far we have been dealing with an integral of the form  a f  x  dx where the interval [a,b] is
finite. In this section we consider an integral defined over an interval that is not finite in length
or one that has a finite number of discontinuities in the interval [a,b].

Definition 1.1.1

Integrals that satisfy either the properties

(a) One or both of the limits of integration are infinite, or


(b) f has a finite number of discontinuities in the interval [a,b] are called improper integrals.

1
1
Consider the area of the region under the graph of y  on the interval 1,  
x2
(see figure 1a).
y

1
y
x2

1a x
Fig 1a

This is of infinite extent which we have not considered before. Now consider the area in figure
1a above but say on a finite interval 1 to b (see figure 1b below).

1
y
x2

b x
1
Figure 1 b

b dx  1   1   1 
b

We evaluate area under the curve as  1 x 2


      
 x 1  b   1 
1
 1
b ____________ (1)

To determine area under the curve in figure 1a we let b approach infinity in equation 1. Thus
 dx b dx
1 x2 b1 x2
 lim

 1
 lim 1  
b  b

=1–0=1

This is an example of an infinitely long region with a finite area.

2
1
Now consider finding the area of the region under the graph of y  on the interval 1,   .
x
(See figure 2a)
y
y
1
y 1
x2 y
x2

1a x 1 b x

2a 2b
To evaluate area in figure 2a, first we evaluate area in 2b given by
k dx
1 x  ln x 1  ln k  ln1
k

 ln k _______________ (2)

and by use of equation 2 we have

 dx k dx
1 x k  1
 lim
x
 lim ln k  
k 

which increases indefinitely hence limit does not exist. This gives an example of an infinitely
long region with an infinite area.

Example 1.1.2

The integrals given by

 dx  dx
1 x
and 
0 x2
are improper because one of the limits of integration is infinite.

Example 1.1.3

The integrals given by

1 dx 3 dx
 0
1 x
and 3  x  1 2

are improper because the integrals have discontinuities somewhere in the interval of integration.

3
For example the integral

1 dx
0 1 x
has a discontinuity at x =1 and hence not integrable there. Thus

1 dx m dx
0  lim 
1  x m1 0 1  x
m

 1
 lim 2 1  x  2 
m1  
0

 1 

 lim 2 1  m  2  2 
m1  

1
= 2 1  1 2  2  0  2  2 .

1.2. Improper Integrals with Infinite Limits of Integration

We define improper integrals with infinite limits of integration as follows.

Definition 1.2.1

 a,   ,
(a) If f is continuous on the interval  then
 b
a f  x  dx  blim
  a
f  x  dx

 , b  ,
(b) If f is continuous on the interval  then
b b
 f  x  dx  k 
lim  f  x  dx
k

 ,   ,
(c) If f is continuous on the interval  then

 c  c b
 f  x  dx   f  x  dx   f  x  dx  lim f  x  dx  lim
  f  x  dx
 c k  k b c
Where c is any real number.

In each case, if the limit exists, then the improper integral is said to converge; otherwise, the
improper integral diverges. In case (c) above, the improper integral on the left diverges if either
of the improper integrals on the right diverges.

We illustrate each of the three cases by use of the following examples

4
Example 1.2.2.

Find
 2 x
0 2e dx

Solution

We notice that this example takes form (a) above. Thus


 2 x b 2 x
0 2e dx  lim
b 0
2e dx
b
 lim  e2 x 
b  0

 lim  e 2b
 e0 
b  
 lim 1  e2b 
b  
 
 lim 1  e 
b
 1 
 lim 1   
b  e 
 lim 1  0  1
b

Example 1.2.3

 dx

Evaluate
0 x2  1

Solution

 dx b dx
0 x 1
2
b 0 x
 lim 2
1
b
 lim  tan 1 x 
b 0

 
 lim tan 1 x  tan 1    
b

2
.

Next we give an example of a divergent improper integral.

Example 1.2.4


Evaluate
 0 cos dx.

5
Solution

 b
cos x dx  lim sin x 0
b
0 cos dx  blim
 0 b
 lim sin b  sin 0
b
 lim sin b
b
As b   ,sin b oscillates between -1 and 1, hence sin b does not approach any number we may
call its limit. Hence we conclude that the given improper integral diverges.

In the following example, we illustrate on how to use L’Hopital’s Rule to evaluate an improper
integral.

Example 1.2.5

Evaluate

 1
1  x  e x dx

Solution

  x dx .
Given 1 1  x  e
we use integration by parts. Let

u  1  x  and dv  e x dx
du   dx v   e x

then
 b
0 1  x  e dx  blim  1  x  e x dx
x
 1

 lim e x  x  1  lim  e x dx


b b
b 1 b 1

 lim e x  x  1  e x 


b b
b 1 1
 
 lim eb  b  1  0  lim eb  e1
b b
 
 lim be b
 
 eb  lim eb  e1 
b b
1
 lim beb  
b e
 b  1
 lim  b  
b  e  e
______________ (3)

6
Applying L’Hopital’s Rule on equation 3, we have
b  1 1 
 lim  b    lim  b   0.
b  e  e b  e 

Hence
 b 1
1 1  x  e
x
dx  lim  b  
b  e  e
1 1
 0   .
e e
In the next example we show how to evaluate an improper integral with infinite upper and lower
limits of integration

Example 1.2.6

Evaluate the improper integral


 x3
 x4  1 2 dx .
 
Solution
 x3 c x3  x3
  dx   dx   dx
  
   
2 2 c 2
x4  1 x4  1 x4  1

We solve the integral on the right by substitution:

Let

u  x4  1
du  4 x 3 dx
du
 dx .
4 x3
x3 x3
 dx  
du 1
 3   u 2 du
x 
2 4 2
4
1 x u 4x 4
1 1 1 4
 
1
u  x 1
4 4
1


4 x4  1 

7
Hence
C S
 x3  1   1 
 x4  1 2 dx  lim    lim 


  
 
R  4 x 4  1 
 R
S   4 x 4  1 
 C

 1 1 

 lim   
R 4 c  1


4
4 
R 4
 1 

  

 1 1 

 lim   
S  4 S  1


4
 
4 c 1 
4
  
1 1 1
  
4 c 1
4
  
4 c  1 2 c4  1
4
  
Now if we let c = 0, we have
 x3 1 1
 dx 



2 c  1 2  0  1
x 
2 4
4
1
1

2

1.3. Improper Integrals with Discontinuities

The second type of improper integral is one that has a discontinuity at or between the limits of
integration.

Definition 1.3.1

(a) If f is continuous on the interval  a, b  and has a discontinuity at x=b, then


b c
a f  x   clim
b a
f  x  dx.

(b) If f is continuous on the interval  a, b and has a discontinuity at x = a, then


b b
a f  x  dx  lim  c f  x  dx
ca

(c) If f is continuous on the interval  a, b  , except  a, b  at which f has discontinuity, then


b c b c m
a f  x  dx  a f  x  dx   f  x  dx  lim f  x  dx  lim 
 f  x  dx 
c k a k mb c
In each case, if the limit exists, then the improper integral is said to converge. Otherwise,
the improper integral diverges. In case (c) the improper integral on the left diverges if either
of the improper integrals on the right diverges.

We can now give a few examples to illustrate all the three cases above.

8
Example 1.3.2

1 dx
Calculate 0 x

Solution

The integrand has a discontinuity at x = 0, thus

1 dx 1 dx 1 1
 0 x
 lim  
lim
b0 b 2 b0 b
1
x 2 dx 
x
 lim  2  2 b 
b0
 2  lim 2 b  2  0
b0
=2
We now give an example of improper integral with a discontinuity in the interior of the
interval (a , b).

Example 1.3.3

2 dx
Evaluate  1 3x

Solution
The given integrand has a discontinuity at x = 0. Thus we write

2 dx 0 dx 2 dx
 1 3   1 3   0 3
x x x

and from the second integral on the right we have


2 dx 2 dx
 0 x 3  b
lim  3
0 b x
2
 1   1 1 
 lim  2   lim   2 
b0  2 x b b0  8 2b 
 .
and it diverges.

Thus since the second integral diverges, it then follows that the original integral
2 dx
1 x3 also diverges.
The next example is a case of a doubly improper integral as defined in case (c) of
definition 1.3.1.

9
Example 1.3.4

 dx
Evaluate 0 x  x  1

Solution

To evaluate the given integral, we split it at a convenient point (say, x = 1) and


 dx 1 dx  dx
0 x  x  1  0 x  x  1  1 x  x  1
1 c
 lim  2 tan 1 x   lim  2 tan 1 x 
b0 b c 1

     
 2   0  2   2 
4 2 4

1.4 Applications of Improper Integrals

Definition 1.4.1

The present value p0 of an amount p due t years later is found by solving the following equation
p
for 0 ;

p0 ek t  p.
p0  p e k t .
where k is the current interest rate.

Example 1.4.2

The accumulated present value of a continuous money flow into an investment of p dollars per
year from now until time T in the future is given by

 T
0 p ekt dt  lim p ekt dt
T  0
p
 lim
T  k 
1  ekt 
p 1 
 lim  1   kt 
T  k  e 
p

k
The following definition is important in evaluation of example 1.4.4.

10
Definition 1.4.3
If the function given by y  f  x  represents a smooth curve on the interval  a, b , then the arc
length of f between a and b is given by

b 2
S 1   f   x  dx .
a
x  g  y
similarly, for a smooth curve given by , the arc length of g between c and d is given
by
d 2
S 1   g   y  dy .
c
in the following example we make use of improper integrals to evaluate arc length.

Example 1.4.4

Use the formula for arc length to show that the circumference of the circle with equation:
x2  y 2  1 is 2 .

Solution

Consider the circle drawn in the figure below


y
1
y  1  x2

x
-1 1

-1
Fig 3
Now consider the quarter circle given by y  1  x 2 , where 0  x  1 . The arc length of this
quarter circle is given by
2
1 1  x 
1   y  dx  
2
S 1  2 
dx
0 0  1 x 
1 dx b dx
  lim 
0 b1 0
1 x 2 1 x 2

 lim  arc sin x 0 
b
0
b1 2

 .
2

11
Hence, multiplying by 4 we conclude that the circumference of the circle is 4S  2 .

In the next subsection we revisit the second fundamental theorem of calculus and introduce the
mean value theorem of integral calculus.

1.5 The Mean Value Theorem of Integral Calculus and its applications.

Firstly we recall the second fundamental theorem of calculus which is as follows:

Theorem 1.5.1. (Second Fundamental Theorem of Calculus)

Suppose f is continuous on its domain which is an open interval I. Then for each point x = a in
I, the definite integral of f from a to x considered as a function of x is an anti-derivative of f. That
is
d  ax f t  dt   f  x dx.
Theorem 1.5.2 (The Mean-Value Theorem for Integrals)

Let f be continuous on  a, b . Then there is a point c strictly between a and b where the value of
f is equal to its average value,

b
a f  x  dx  f  c b  a  .
Proof
f(x)

f average f(c)

a c b
Fig 4
We can make f continuous on the whole real line by defining f  x   f  b  for x  b. By the
second fundamental theorem of Calculus, f has an anti-derivative F. By the mean value theorem
there is a point c strictly between a and b at which F   c  is equal to the average slope of F,

12
F b   F  a 
Fc 
ba
But
F  c  f c
and
b
F  b   F  a    f  x  dx,
a so
b
F c 
a f  x  dx .
ba

Example 1.5.3

Let f : 1, 3  be defined by f  x   x 2 .

Find the value of c which satisfies the Mean Value Theorem of Integral Calculus for the
function f  x  .

Solution

3
3 2 x3 26
 1
x dx 
3

3
1

We find c such that

26
f c  2 
3
13
 c2 
3
13
c
3
13
of these two values of c, only belongs in the interval 1, 3 . Hence
3
3  13 
1 f  x  dx  f  3  3  1 .
 
The following example provides another situation where Mean Value Theorem can be applied.

Example 1.5.4

A car starts at rest and moves with velocity v  3t 2 . Find its average velocity between times
t  0 and t  5 . At what point of its time  is its velocity equal to the average velocity?

13
Solution

Let Vave denote average velocity.

Then
5 5

Vave 
 0
3t 2 dt

t5
0
50 5
125
  25
5

Now putting
3t 2  25 we obtain

25 5
t  .
3 3

14
1.6. Self Test 1

In questions 1 – 13, determine whether the improper integral is convergent or divergent, and
calculate its value if it is convergent.
 dx  
1.  2
2.  e x dx 3.  x e x dx
3 x 0 0

 2 1 0
4. 1 ln x dx 5. 0 2
dx 6.   e2 x dx
 x  1 3

 1   1
7. 1 x
dx 8. 0 e x cos x dx 9.   1  x2
dx

 1 1  1
10. 0 e x  e x
dx 11. 0 x ln x dx 12. 0 3
8 x
dx

 2 1
13. 0 2 tan  d 14. 0 4  x2
dx

15. Find the area, if it exists of the region under the graph of
y  2 on the interval 
1 2,  
.
x

16. Find the accumulated present value of an investment for which there is a perpetual
continuous money flow of $3600 per year. The current interest is 8%.

17. Plutonium has a decay of 0.003% per year. Suppose plutonium is released into the
atmosphere each year at the rate of 1 pound per year. What is the limiting value of the
radioactive build up?

18. Find c satisfying the mean value theorem for integrals

0 x   2  x 
5 2
(a) 3
 1 dx (b) 3
 1 dx

19. Use the mean value theorem for integrals to prove the inequality


2 dx 1 
 0 x2  4  2  2 cos x dx  3 .
2
(a) (b)
6

20. Use the Second Fundamental Theorem of calculus to find F   x  if

t 
x
F x   2
 2t  5 dt .
2

15
1.7. Self Test 1

b
b  1 1 1

1. 2
x dx         
3  x 3 b 3
1 1  1 1 1
lim       . Converges
b  3 b  3  3

b

2. e x   lim eb  1  . Diverges
0 b 
 u x
3. 
0
xe x dx  dv  e x dx
du  dx vex
b b b
 x e x   e x    x  1 e x 
0 0 0

  b  1 eb  0  1 e0   b  1 eb  1  
diverges.

u  ln x dv  dx
b
4.  1
ln x dx 
du 
dx
vx
x
dx
  x ln x     x   x ln x 1  x 1   blub   ln1  b  1
b b
x
 b lub b  1   . Diverges

5. Converges to 6

0 1  2 x  0 1  0 2a  1 
6. a e2 x dx  e  e e  1  e 2a 
2   0 2   2 

1  e2   . Converges
1 1

2  2

b
1 1

b 
7. x 2 dx  2 x 2   2  b  1   Diverges
1

1
1  1
8. 9.  10. 11.  12. 6
2 4 4

13. Diverges

16
2 1 x  2 sin 
14.  0
dx
dx  2 cos  d
4  x2
1 1 
 dx    2cos d 
4  x2 4  4sin 2  2

sin 1 1  sin 1  0   0.
2
 x
d    sin 1  
2


2
0
 1 1  1 1
15.  2 2
x
      
 x 2   2 
1

2

P 36000
16.   45000
R 0.08

17.

5
 1  x4  125 105
18. (a) 2 2
    x  5   5 fcc)
x  4  0 4 4
105 21
c3  1  
5x 4 4
21 4 25
c3   
4 4 4
21
c3
4

(b) f  x   x3  1  c3  1

2
 x4 
  x
 4 
3   2  4  2    4  2 
c 1   1
4 4
c3  0  c  0

17
x
 t3 2 
  t  5t 
 3 
  2  x3   8 
20.   x 2  5 x     4  10 
4  3   3 
 
x3  8  12  30 
  x2  5x   
3  3 
x3 50
  x2  5x 
3 3

18
LECTURE 2
FUNCTIONS OF SEVERAL VARIABLES AND APPLICATIONS

2.0 Introduction

The types of functions we have so far considered are of single variables. In this section we
introduce functions of several variables and more so consider some of their properties. We also
look at some of their applications in real life situations, and other areas like engineering,
commerce and industry.

Firstly, we consider the force work equation given by Work = Force x Distance and the volume
of a right circular cylinder given by V   r 2 h.
We observe that the above equations are both functions of two variables.

Objectives

At the end of this lecture you should be able to:


Define domain and Range of functions of two and three variables
Graph functions of two and three variables
Define limit of a function of two and three variables
Solve problems on limit of functions of several variables
Define continuous functions of several variables
Identify continuous functions

2.1 Definition of Domain and Range of Functions of Several Variables

Let D be a set of ordered pairs of real numbers. If to each ordered pair (x , y) in D there
corresponds a real number f  x, y  , then f is called a function of x and y.

The set D is the domain of f, and the corresponding set of values for f  x, y  is the range of f.
For the function z  f  x, y  , we call x and y the independent variables and z the dependent
variable. Similar definitions can be given for functions of three, four or n variables, where the
domains consist of ordered triples  x1 , x2 , x3  , quadruples  x1 , x2 , x3 , x4  and n-tuples
 x1 , x2 ,..., xn 
respectively. We restrict to studying functions of two and three variables and the
domain is the set of all points for which the equation is defined. For instance the domain of the
function given by

f  x, y   x 2  y 2
is assumed to be the entire xy – plane.

19
Example 2.1.1.

The total cost of a company, in thousands of Kenya Shillings, is given by

c  x, y, z   3x 2  5xy  z

where kshs. x is spent for labour, kshs. y for raw materials and kshs. z for advertising. Find
c 8, 4, 2  .

Solution

c  8, 4, 2   3 8   5 8  4  2
2

 192  160  2
 kshs.354

In the next example, we show how to determine range and domains of certain functions.

Example 2.1.2

Find the domains and ranges for the following functions

x2  y 2  9
(a) f  x, y  
x

x
(b) g  x, y, z  
9  x  y2  z2
2

Solution

(a) We see that the function given by

x2  y 2  9
f  x, y  
x

is defined for all points  x, y  such that x  0 and x 2  y 2  9. Thus the domain is the set
of all points lying on or outside the circle x 2  y 2  9, except those on the y-axis (see figure
below)

20
Hollow circle
y
3

Hollow circle

x
-3 3

-3

The range of f  x, y  is such that f  x, y   0 .

x
(b) The function g  x, y, z  
9  x2  y 2  z 2
can be rewritten as

x
g  x, y , z  

9  x2  y 2  z 2 
for g to be defined then the denominator should not be zero and more so
 
9  x 2  y 2  z 2 so as to obtain real roots. Thus

x2  y 2  z 2  9

Consequently, the domain is the set of all points  x, y, z  lying inside a sphere of radius 3
that is centered at the origin (see figure below).
z
3

-3 y

3 -3
x

The range of g  x, y, z  is such that g  x, y, z   0.

21
The following table gives domains and ranges of various functions of two and three
variables.

Table 1

Function Domain Range

W  y  x2 y  x2 W 0
1
W
xy xy  0 W 0
W  sin xy Entire Plane 1  W  1

1
W
x  y2
2
 x, y    0, 0   W  0

W  x2  y 2  z 2 Entire space W 0

1
W
x  y2  z2
2
 x, y, z    0,0,0 0 W  

Definition 2.1.3 Composition

Functions of several variables can be combined in the same ways as functions of a single
variable. For two variables, we have
 f  g   x, y   f  x, y   g  x , y 
 f  g   x, y   f  x, y   g  x, y 
 fg   x, y   f  x, y   g  x, y 
 f  f  x, y 
   x, y   provided g  x, y   0
g g  x, y 

The composite function given by  goh   x, y  is defined only if h is a function of  x, y  and g is


a function of a single variable. Then

 goh  x, y   g  h  x, y  

for all  x, y  in the domain of h such that h  x, y  is in the domain of g. For instance, the
function given by
f  x, y   9  2 x 2  y 2 .

22
can be viewed as the composite of the function of two variables given by
h  x, y   9  2 x 2  y 2 and the function of a single variable given by g  u   u . Therefore
 goh  x, y   g  h  x, y    h  x, y 

 9  2 x2  y 2

Remark

The domains of functions defined on portions of the plane can have interior points and boundary
points just as the domains of functions defined on intervals of the real line can. The figure below
illustrates the two points

R
R

 x0 , y0 

 x0 , y0 
Fig 2a Fig 2b
Interior point boundary point

Definition 2.1.4

A point  x0 , y0  in a region R in the xy – plane is an interior point of R if it is the centre of a disk


that lies entirely in R.

A point  x0 , y0  is a boundary point of R if every disk centered at  x0 , y0  contains points that lie
outside of R as well as points that lie in R. Notice that the boundary point itself need not belong
to R.

The interior points of a region, as used, make up the interior of the region. The region’s
boundary points make up its boundary. A region is said to be open if it consists entirely of
interior points. A region is said to be closed if it contains of all its boundary points. The figure
below shows all the three cases of a unit disk.

23
y y y

x x x
0 0 0

Fig 2a Fig 2b Fig 2c


 x, y  x2  y 2  1  x, y  x2  y 2  1  x, y  x2  y 2  1
Open unit disk every point an Boundary of unit disk closed unit disk. Interior
point (the unit circle) contains all Boundary
Points.

2.3 Graphing of a function of several variables

In this subsection we consider graphing of a function of two variables. It is imperative that you
understand the domains and ranges of functions of several variables before attempting this
subsection.

Consider a function of two variables

z  f  x, y  .

As a mapping, a function of two variables can be thought of as mapping a point  x1 , y1  in an xy-


plane onto a point z1 on a number line.
y

y1 (x1,y1) z1  f  x1 , y1 

x
x1
To graph a function of two variables, we need a three dimensional coordinate system. The graph
of a function of two variables is the set of all points
 x, y, z  for which z  f  x, y  and  x, y  is
the domain of f. The line z, called the z-axis is placed perpendicular to the xy – plane at the
origin.

24
z

z1 (x1,y1,z1)

, y1 y

x1
(x1,y1)
x

In the figure above we demonstrate how to plot a point  x1 , y1 , z1  . First to locate the point
 x1 , y1  , identify the point x
and from it trace a line parallel to the y-axis and similarly from the
1

point y1 trace a line parallel to the x-axis. The point where they meet gives the coordinate point
 x1 , y1  .
Now from the point  x1 , y1  trace a ray z1 steps upwards parallel to the z-axis. The new point
located gives the coordinate point  x1 , y1 , z1  .

Example 2.3.1

Plot the points p1  2, 3, 5 , p2  2,  2,  4  , p3  0, 5, 2  and p4  2, 3, 0 

Solution z

5
P1(2,3,5)
4
3
2 P3(0,5,2)
1
y
-5 -4 -3 -2 -1 1 2 3 4 5
1
2 -2 P4(2,3,0)
3 -3
4 -4
5 -5
P2(2,-2,4)

The graph of a function of two variables z  f  x, y  consists of ordered triples

25
 x1 , y1 , z1  , where z1  f  x1 , y1  . The point  x1 , y1  is in the domain of f.

Example 2.3.2

Plot the planes x = 4 and y = 5. Also plot z  x 2  y 2 .

Solution
z
z

y=5

x=4 y

y
x

x
z

Elliptic Paraboloid

z  x2  y 2

Example 2.3.3
Sketch the graph of f  x, y   16  4 x 2  y 2 . What is the range of f?

Solution

 
The domain D of the function f  x, y   16  4 x 2  y 2  16  4 x 2  y 2 is the set of all points
such that 16  4 x  y OR 4 x  y  16 which simplifies to
2 2 2 2

x2 y2
  1.
4 16

26
The range of f is all values z  f  x, y  such that 0  z  16  4. A point  x, y, z  is on the
graph of f if and only if

z  16  4 x 2  y 2
z 2  16  4 x 2  y 2
4 x 2  y 2  z 2  16
x2 y2 z2
   1for 0  z  4.
4 16 16

This equation represents the upper half of an ellipsoid as shown in the figure below

z
4 Surface: z  16  4 x2  y 2

Trace in plane z = 2 Range

y
4

2
Domain

x
Graph of f  x, y   16  4 x 2  y 2

Remark

To sketch a surface in space, it is helpful to use traces in planes parallel to the coordinate planes,
as shown in the figure above. For example to find the trace of the surface in the plane z  2, we
substitute z  2 in the equation z  16  4 x 2  y 2 and obtain

x2 y 2
 1
3 12

which gives an equation of ellipse centered at the point  0, 0, 2  with major and minor axes of
lengths 4 3 and 2 3 , respectively.

27
2.4. Limit of a function of two variables

Using the formula for the distance   0 between two points  x, y  and  x0 , y0  in the plane, we
define the   neighbourhood about  x0 , y0  to be the disk centered at  x0 , y0  with radius  :

  x, y  :  x  x0 
2
  y  y0 
2
 
as shown in the figure below

 x, y 
 Open disk

 x0 , y0 

With the less than inequality the disk is open and with less or equal inequality the disk is closed.

Definition 2.4.1


Let f be a function of two variables defined, except possibly at x0 , y0 , and let L be a real 
number. Then
lim f  x, y   L
 x, y   x0 , y0 

if for each  0, there corresponds a   0 such that

 x  x0    y  y0 
2 2
f  x, y   L  whenever 0  

Remark

 
In a function of two variables, when we write  x, y   x0 , y0 we mean that the point  x, y  is
 
allowed to approach x0 , y0 from any direction. If the value of lim
 x, y   x0 , y0 
f  x, y  is not the

same for all possible approaches or paths, to  x0 , y0  , then the limit does not exist.

28
Example 2.4.2

Show that lim xa


 x, y   a,b 

Solution

Let f  x, y   x and L  a. We need to show that for each  0, there exists a


  neighbourhood about  a, b  such that

f  x, y   L  x  a 

whenever  x, y    a, b  lies in the neighbourhood. We observe that from

 x  a   y  b  
2 2
0

if follows
f  x, y   a  x  a   x  a  x  a   y  b
2 2 2

Thus, we can choose  , and the limit is verified.

Example 2.4.3

Evaluate

sin xy x2 y
(a) lim (b) lim  0.
 x, y   2,  1  xy
   x, y   0,0 x 2  y 2
 4 

Solution
  
(a) lim sin xy  sin  2 x   sin  1
 
 x, y   2,

  4 2

 4 

  2
lim 1  xy  1  2  1  .
   4 2 2
 x, y   2, 

 4 

we know that limit of a quotient equals to the quotient of the limit, so

sin xy 1 2
lim  
 x, y   22  1  xy  2    2  
 
 2 

29
(b) We first note that

x2
y x  y and 2
2 2
 1.
x  y2

x2 y
Let f  x, y   , L  0 and  a, b    0, 0 
x2  y 2
We need to show that for each  0, there exists a   neighbourhood about (0,0) such that
x2 y x2 y
f  x, y   L   0   .
x2  y 2 x2  y 2

Whenever  x, y    0, 0 lies in the neigbourhood. Then it is a   neighbourhood about (0,0),


we have 0  x 2  y 2   , and it follows that for  x, y    0, 0  ,
x2 y x2  x2 
f  x, y   0   y  y  2 2 
x2  y 2 x2  y 2 x y 
 y  x2  y 2   .

Thus we choose   and conclude that


x2 y
lim 0
 x, y   0,0 x 2  y 2

Example 2.4.4

Show that the following limit does not exist


2
 x2  y 2 
lim  
 x, y   0,0  x 2  y 2 

Solution

The domain of the function given by


2
 x2  y 2 
f  x, y    2 2 
x y 

is defined for of all points in the xy-plane except for the point (0,0). To show that the limit as
(x,y) approaches (0,0) does not exist, we consider approaching along two different paths.
Along the x-axis, every point is of the form (x,0) and the limit along this approach is
2
 x2  0 
lim 1  1
2
lim  2  
 x,0  0,0   x  0   x,0   0,0 

30
However, if  x, y  approaches (0,0) along the line y = x, we obtain
2
 x2  x2 
2
 0 
lim  2 2 
 lim    0
 x, y   0,0  x  x   x, x   0,0   2 x 

This means that in any open disk centered at (0,0) there are points (x,y) at which f takes
on the value 1. And other points at which f takes on the value 0. Hence f does not have a
limit as  x, y    0, 0  .

Example 2.4.5
x3  y 3
Show that lim does not exist.
 x, y   0,0 x3  y 3

Solution

If  x, y    0, 0  along the x-axis, (i.e. along y =0), then

x3  y 3 x3
lim  lim 1
 x, y   0,0 x3  y 3  x, y   0,0  x3

On the other hand let  x, y    0, 0  along the y-axis (i.e. along x = 0), then
x3  y 3  y3
lim  lim  1
 x, y   0,0  x3  y 3  x, y   0,0  y 3

Since the limiting values are different along the two paths, the limit does not exist.

2.5 Continuity of functions of two and three variables.

Definition 2.5.1

 
A function f of two variables is called continuous at a point x0 , y0 if the following conditions
are satisfied.
1. f  x0 , y0  is defined
2. lim f  x, y  exists
 x, y   x0 , y 0 
3. lim f  x, y   f  x0 , y0 
 x, y   x0 , y 0 

A function of two variables is called continuous on a region R of the xy-plane if it is continuous


at each point of R. A function that is continuous on the entire xy-plane is called continuous
everywhere or simply continuous.

31
How does one identify a continuous function? The answer is found in the following theorem.

Theorem 2.5.2

(a) If g and h are continuous functions of one variable, then f  x, y   g  h  x, y   is a


continuous function of x and y.

(b) If g is a continuous function of one variable and h is a continuous function of two variables,
then their composition f  x, y   g  h  x, y   is a continuous function of x and y.

Example 2.5.3

Since g  x   x2 and h  y   2 y3 are continuous functions, it follows from part (a) of


theorem 2.4.7 that

f  x, y   2 x2 y3
is a continuous function of x and y.

Example 2.5.4

Since g  x   cos x is continuous, and g  x, y   2 x2 y3 is continuous by example 2.4.8, it


follows from part (b) of theorem 2.4.7 that

  
g  h  x, y    g x 2 y3  cos x 2 y3 
is a continuous function of x and y.

By a similar argument show that each of the following is continuous

1 
x

(a) f  x, y   x y 3 2
 5 (b) f  x, y   e y
, y0

(c) f  x, y   cos h  x  y  .

Example 2.5.5
Discuss the continuity of the following functions

x2 2
(a) f  x, y   (b) g  x, y  
1 y2 y  x2

32
Solution

(a) Since a rational function is continuous at every point in its domain, we conclude that f is
continuous at each point in the xy- plane.

(b) The function given by


2
g  x, y  
y  x2
is continuous everywhere except at the points at which the denominator is zero, y  x 2  0.
Thus, we conclude that the function is continuous at all points except those lying on the
parabola y  x 2 . Inside this parabola, we have y  x 2 , and the surface represented by the
function lies above the xy-plane. Outside the parabola, y  x 2 , and the surface lies below
the xy-plane.

Remark

The sum, difference and product of continuous functions of two variables are also
continuous. Moreover, the ratio of continuous functions is continuous except where the
denominator is zero.

Example 2.5.6

xy
Evaluate lim
 x, y   2,3 x  y 2
2

by use of definition of continuity.

Solution

xy
Since a rational function f  x, y   is continuous, the limit is the same as the
x2  y 2
value of f at (-2,-3).

xy  2 3 6
Thus lim  
 x, y   2,3 x  y  2   3 13
2 2 2 2

In the following example we make use of definition of continuity in terms of limit to


show that the given function is not continuous at (0,0).

33
Example 2.5.7

Show that the function

 xy
 2 ,  x, y    0, 0 
f  x, y    x  y
2

 0 ,  x, y    0, 0 

is continuous at every point except the origin.

Solution

The function f is continuous at any point  x, y    0, 0  because its values are then given
by a rational function of x and y. At (0,0) the value of f is defined, but f, we claim has not
limit as  x, y    0, 0  . For every value of m, the line y  mx is a level curve of f
because the value of f on the line has the constant value,

xy
f  x, y  
x 2  y 2 y mx

x  mx  m
 
x 2   mx  1  m2
2

Therefore, f has this number as its limit as  x, y    0, 0  along the line y  mx :

lim f  x, y   lim  f  x, y  
 x, y   0,0  x, y   0,0   y mx 
along y  mx .

 xy  m
 lim  2  2
 x, y   0,0   x  y y mx  1  m2
2
 
The limit changes with m. For instance, along the line y  x, m  1 and

1 1
 lim f  x, y   
 x, y   0,0 1  1
2
2
along y  x

1 1
while along the line y  x , m  and
4 4

34
1
 
4 4
lim f  x, y   
 x, y   0,0  1
2
17
1  
4
there is therefore no single number we may call the limit of f as  x, y  approaches the
origin. The limit fails to exist, and the function is not continuous.

Example 2.5.8
x2 y
Given f  x, y  
x4  y 2
Find lim f  x, y  if it exists.
 x, y   0,0

Solution

The function f is defined everywhere in R2 except at (0,0). Along the curve y  kx 2 , x  0, the
function has the constant value

f  x, y  y kx2 
x 2 kx 2

k  
1 k 2  
2
x 4  kx 2
therefore,
lim f  x, y   lim  f  x, y  
 x, y   0,0  x, y   0,0  y kx2 

along y  kx 2
k

1 k 2
This limit varies with the path of approach. For instance if  x, y    0, 0  along the parabola
1
y  x 2 , k  1, and the limit is .
2

If  x, y    0, 0  along the x-axis, k = 0 and the limit is 0. Thus there is no single number we
may call the limit of f, and by the two-path test we can conclude that f has no limit as
 x, y    0,0 .

Remark
The definitions of limit and continuity for functions of two variables and the conclusions
about limits for sums, products, quotients and composites of functions of two variables
all extend to functions of three or more variables. Thus, functions like
y 2 sin x
ln  x  y  z  and
z 1

35
are continuous throughout their domains and limit of say a function of three id given by
xz
e e11
lim 
 x, y, z  1,0,0  z 2  cos xy  1
2
 cos 0
1

2
Definition 2.5.9

A function f of three variables is continuous at a point  x0 , y0 , z0  in an open region R if


 
f x0 , y0 , z0 is defined and equal to the limit of f  x, y, z  as  x, y, z  approaches
 x0 , y0 , z0  . That is
lim f  x, y, z   f  x0 , y0 , z0  .
 x, y, z   x0 , y0 , z0 

The function f is continuous in the open region R if it is continuous at every point in R.

Example 2.5.10

sin xy
The function f  x, y, z  
x  y2  z
2

is continuous at each point in space except at the points on the paraboloid given by z  x 2  y 2 ,
since the function fails to exist when x2  y 2  z  0 .

36
2.6 Self Test Questions

In question 1 – 4 evaluate the given function at the indicated point

x 1
1. f  x, y  
y2  x
(a) (2, 4) (b) (-1,-2) (c) (0, -4)

2. f  x, y   x 2 e y

(a) (2, 3) (b) (-2,-2) (c) (-3, 0)

3. f  x, y, z   x 2  y 2  z 2

(a) (0, 0, 0) (b) (0, 2, 3) (c) (-1, -1, -1)

y dt
4. h  x, y   
x t
(a) (4, 1) (b) (6, 3)

5. The Doyle Log Rule is used to determine the lumber yield of a log (in board-feet) in terms of
its diameter d (in inches) and its length L (in feet). The number of board-feet is given by
2
 d 4
N d, L    L
 4 

(a) Find the number of board-feet of lumber in a log 22 inches in diameter and 12 feet in
length.

(b) Find N (30, 12)

For questions 6-8, describe the region R in the xy-coordinate plane that corresponds to the
domain of the given function and find the range of the function.

6. f  x, y   4  x 2  y 2 7. f  x, y   cos1  x  y 

x y
8. z 
xy

9. Sketch the level curves


(i) 3x  y  2 (ii) x2  4 y 2  4

37
in questions 10 -13 find the indicated limit and discuss the continuity of the function
x
arc cos  
y  y
10. lim 11. lim
 x, y  3, 6 x  y  x, y   0, 2 1  xy

x

y
e 1
12. lim 13. lim
 x, y, z   0, 2,1 x  y 2  z
2
 x, y, z   1, 1, 2 x 2  y 2  z 2

In questions 14-18 evaluate the limit

x 2  3x xy  2 x
14. lim 15. lim
 x, y   0, 0 4 x  xy  x, y   1, 2 xy  6  2 x  3 y

x 2  2 xy  y 2 y4
lim lim
16.  x, y  1, 1 x y 17.  x, y   2, 4 x y  xy  4 x 2  4 x
2

x y y  4, x  x
2

18. lim tan 1  xyz 


 x, y, z  1, 1, 1

In questions 19-21 determine the limit if it exists

3x 2  y 2  xy 2
19. lim 20. lim
 x, y   0, 0 x 2  3 y 2  x, y   0, 0 x 2  y 4

x4  y 4
21. lim
 x, y   0, 0 x 2  y 2

in questions 22-23 discuss the continuity of the composite function fog.

1
22. f  t   t 2 , g  x, y   3x  2 y 23. f  t   , g  x, y   3x  2 y.
t

at what points in the plane are the functions in questions 24-25 continuous

24. (a) sin  x  y  


(b) ln x 2  y 2  1 
1 1
25. (a) cos (b)
xy x y
2

38
at what points in space are the functions in questions 26-27 continuous.

26. (a) x 2  y 2  2 z 2 (b) x2  y 2  1

1 1
27. (a) (b)
x yz xy  z

39
2.7 Solutions to Self test 2.6

2 1 1 1  1 2
1 (a) f  2, 4    (b) f  1, 2   
16  2 14 4 1 5
0 1 1
(c) f  0, 4   
16  0 16

2. f  x, y   x2e y

(a) f  2,3  4e 3 (b) f  2, 2   4e 2 (c) f  3, 0   9e 0  9

3. f  x, y, z   x 2  y 2  z 2

(a) f  0,0,0   0 (b) f  0, 2,3  13 (c) f  1, 1, 1  3

y dt
4. h  x, y   
x t

(a) h  4,1  int 4  ln1  ln 4   ln 4 (b) h  6,3  ln 3  ln 6   ln 2


1

5 (a) 243 board feet (b) 507 board feet

6. f  x, y   4  x 2  y 2 : R : 4  x 2  y 2
circle with radius 2 lies on or inside.
Range: f  x, y   0.

7. f  x, y   cos1  x  y 
Domain: R :   x, y  : 1  x  y  1 
Range:   f  x, y   0 .

8. Domain:   x, y  : x  0, y  0 
Range: All real numbers

40
9. (i) 3x – y = 2
z
2

y
-2 -1 0 1 2

x 3x  y  2

(ii) x2  4 y 2  4
z

-2

y
-1 0 1

2 x2  y 2  4
x

y 6
10. lim  2
 x y  36 
x y 3
continuous everywhere except at x = -y.

x
cos 1  
11. lim  y   cos 1  0    .
 x y  0,2  1  xy 2
x
continuous for xy  1, y  0, 1 .
y
x
y
y e0 1
12. lim  
 x, y , z  0,2,1 x 2  y 2  z 0  4  1 3

continuous for z  x2  y 2  z and y  0.

1 1 1
13. lim  
 x, y, z  1,1,2 
x2  y 2  z 2 11 4 6
continuous everywhere except at the origin (0,0,0)

41
x 2  3x x  x  3 3
14. lim  
 x, y  0,0  4 x  xy x  4  y  4

xy  2 x x  y  2 x  y  2
15. lim  
 x, y  1,2 xy  6  2 x  3 y xy  2 x  3 y  6 x  y  2   3  y  2 

x 1 1
  
 x  3 1  3 2
x 2  3xy  y 2  x  y 
2
16. lim   x y 0
 x, y 1,1 x y x y

y4 y4 1
17. lim  
 x, y  2,4 x 2 y  xy  4 x 2  4 x x 2  y  4   x  y  4  2


18. lim tan 1  x, y, z   tan 1 1 
 x, y, z 1,1,1 4

3x 2  y 2
19. lim  lim
 x, y  0,0  x 2  3 y 2  0

20.

r cos  r 2 sin 2   0
r 2 cos2   r 4 cos4 


    


2
21.  x 2  y 2  x 2  y 2  0
 

22. f  t   t 2 , g  x, y   3x  2 y
f g  x, y    3x  2 y   CONTINUOUS
2

1
23. f  t   , g  x, y   3x  2 y
t
1 3
f g  x, y   ; CONTINUOUS except y  x.
3x  2 y 2

24. (a) f  x, y   sin  x  y  All (x,y)

 
(b) ln x 2  y 2  1 : All (x,y) except those using on cylinder x2  y 2  1

42
 
25. (a) f  x, y  cos1  1  . All except x = 0, y = 0
xy
1
(b) f  x, y  cos1 2 ; All (x,y) except those lying on parabola y  x 2 .
x y

26. (a) f  x, y, z   x2  y 2  2 z 2 : All (x,y,z) or continuous everywhere.

(b) f  x, y, z   x 2  y 2  1 : All (x,y,z) except on or insider cylinder x2  y 2  1 .

1
27. (a) f  x, y, z   Everywhere except origin (0,0,0).
x yz

1
(b) f  x, y, z   Everywhere expect xy = 0 and z = 0.
xy  z

43
LECTURE 3

LAGRANGE’S MULTIPLIERS

3.0 Introduction

In this section we survey a few of the many applications of the theory of maxima and minima for
functions of which several variables was well covered in calculus III.

Objectives

At least end of this lecture you should be able to:

 Solve optimization problems


 State and proof the theorem on Least Squares Regression Line
 State and proof Lagrange’s Theorem
 Apply Lagrange’s theorem to solve optimization problems

3.1 Applied Optimization Problems

In this subsection we consider optimizations problems free of constraints.

Example 3.1.1

A rectangular box is resting on the xy-plane with one vertex at the origin. Find the maximum
volume of the box if its vertex opposite the origin lies in the plane.

6 x  4 y  3z  24,
as shown in the figure below.
z
0, 0, 8 plane
6 x  4 y  3z  24

0, 6, 0 y

4, 0, 0
x

44
Solution

Since one vertex of the box lies in the plane 6 x  4 y  3z  24, solving for z we have
1
Z   24  6 x  4 y 
3
and the volume xyz of box can be written as a function of two variables:
1 
V  x, y    x  y    24  6 xy  4 y 
3 
1

 24 xy  6 yx 2  4 xy 2
3

1

Vx  x, y   24 y  12 xy  4 y 2
3

y
  24  12 x  4 y 
3
1

Vy  x, y   24 x  6 x 2  8 yx
3

x
  24  6 x  8 y  .
3

4 
Equating Vx  x, y   0 and Vy  x, y   0 , we obtain critical points to be (0,0) and  , 2  . At
3 
 0, 0  the volume is zero, so ignore the point. Thus we apply the second partials tests to the
4 
point  , 2 .
3 

8 x
Vxx  x, y   4 y, V yy  x, y  
3
and
1
Vxy  x, y    24  12 x  8 y 
3

By the second partials test

2
4  4   4 
Vxx  , 2  V yy  , 2   Vxy  3 , 2  
3  3    
2
 32   8  64
  8     0
 9   3 3
and
4 
Vxx  , 2   8  0.
3 

45
Thus the maximum volume is

2
4  1 4  4 2  4 
2
4
Vxx  , 2    24    2   6    2   4    2    Vxy  3 , 2  
 3  3   3  3 3    
64

9 cubic units.

In many applications of extrema, we often involve more than one independent variable. The next
example illustrates an application involving two products.

Example 3.1.2

The profit obtained by producing x units of product A and y units of product B is approximated
by the model

 
p  x, y   8x  10 y   0.001 x 2  xy  y 2  10000

find the production level that produces a maximum profit.

Solution

Differentiating p  x, y  partially with respect to x and y we obtain


px  x, y   8   0.001  2 x  y  and
p y  x, y   10   0.001  x  2 y  .

setting px  x, y   0 and p y  x, y   0 we obtain

8   0.001  2 x  y   0  2 x  y  8000
10   0.001  x  2 y   0  x  2 y  10000

On solving the equations on the right simultaneously we obtain

x = 2000 and y = 4000

Differentiating px  x, y  again partially with respect to x and y we obtain


pxx  x, y   0.002
p yy  x, y   0.002 and
pxy  x, y   0.001

46
Substituting in values of x = 2000 and y = 4000 we obtain
pxx  2000, 4000   0.002
p yy  2000, 4000   0.002
pxy  2000, 4000   0.001
Moreover, since pxx  0 and
pxx  2000, 4000  p yy  2000, 4000    pxy  2000, 4000    0.001
2

  0.002    0.001  0.
2 2

Thus the production level of x  2000 units and y = 4000 units yields a maximum profit.

Remark

In example 3.1.2 we assumed that the plant is able to produce the required number of units to
yield a maximum profit. In actual practice, the production would be bounded by physical
constraints. In such cases we employ the method of least squares.

3.2. The Method of Least Squares

In this subsection we look a method for obtaining mathematical models. Consider plotting the
points (2,1), (5,2), (7,6), (9,12) and (11, 17)
y y

17 (11,17) 17 (11,17)
16 16
15 15
14 14
13 13
12 12 (9, 12)
(9, 12)
11 11
10 10
9 9
8 8
7 7
6 (7, 6) 6 (7, 6)
5 5
4 4
3 3
2 2 (5, 2)
(2, 1)
1 (2, 1) 1
x x
0 6 8 10 12 0 6 8 10 12
2 4 2 4
Fig 3b Fig 3c

47
A simple linear model for the points in figure 3b is

y 1.8566 x  5.0246

Fig 3c gives a slightly more complicated quadratic model of

y  0.1996 x2  0.7281x  1.3749

which achieves greater accuracy.

As a measure of how well the model y  f  x  fits the collection of points

  x , y , x
1 1 2 , y2  ,  x3 , y3  ,...,  xn , yn  

We add the squares of the differences between the actual y-values and the values given
by the model to obtain the sum of the squared errors

n
S    f  xi   yi 
2

i 1 .

Graphically, S can be interpreted as the sum of the squares of the vertical distances
between the graph of f and the given points in the plane, as shown in figure 3d

 x1, y1 
 x3 , y3 
d1

d3
d2
 x2 , y2 
Fig 3d S  d 12  d 22  d 23

If the model is perfect, then S=0. However should S  0 , then we settle for a model that
minimizes S. Such a model is called the least squares regression line.

Theorem 3.2.1

The least squares regression line for  x , y ,x


1 1 2 , y2  ,...,  xn , yn   is given by
f  x   ax  b where

48
n n n
n xi yi   xi  yi
a  i 1 i 1 i 1
2
n   n
n x    xi 
2
i
i 1  i 1  and
1 n  n
b    yi  a  xi 
n  i 1 i 1 

Proof

Let S  a, b  represents the sum of the squared errors for the model
f  x   ax  b

at the given set of points. That is,


n n
S  a, b     f  xi   yi    axi  b  yi 
2 2

i 1 i 1

where the points  xi , yi  represent constants.


n
Sa  a, b   2 xi  axi  b  yi 
i 1
n n n
 2a x12  2b xi  2 yi
i 1 i 1 i 1
n
Sb  a, b   2 xi  axi  b  yi 
i 1
n n
 2a xi  2nb  2 yi
i 1 i 1

At the extremum points, Sa  Sb  0 which gives


n n n
2a xi2  2b xi  2 xi yi 0
i 1 i 1 i 1 __________ 1
and
n n
2a xi  2nb  2 yi  0 __________ 2
i 1 i 1

Solving the second equation for b yields

nb   yi  a  xi
1 n n 
b  i y  a  xi 
n  i 1 i 1 

49
and substituting this value of b in equation 1 we obtain

1 n 2 1 n  n n  n
a 
n i 1
xi   xi   yi  a xi    xi yi  0
n i 1  i 1 i 1  i 1

which on solving for a yields

n n n
n xi yi   xi  yi
a  i 1 i 1 i 1
2
n  n

n xi2    xi 
i 1  i 1 

By the second partials test we have:

n
Saa  a, b   2 xi2
i 1 _________ 3
and
Saa  a, b   2n

while
n
Sab  a, b   2 xi
i 1

Since

 
2
Sab  a, b  Sbb  a, b   Sab  a, b 
2
 n   n 
  2 xi2  2n   2 xi 
 i 1   i 1 
2
n  n 
 4n xi  4   xi   0
2
i 1  i 1 
and

n
Saa  a, b   2 xi2  0
i 1

We conclude from the second partials test that the values (a, b) yields a minimum.

If the values of x are symmetrically spaced about the y-axis, then  xi 0 and the
formulas for a and b simplify to

50
n
n xi yi
1 n
a  i n1 and b  y
n i 1 i

i
xi2
1

Example 3.2.2

Find the least squares regression line for the following points
(-3, 0), (-1, 1), (0, 2), (2, 3)

Solution

The table below shows the calculations for finding the least squares regression line using
n=4

x y xy x2
-3 0 0 9
-1 1 -1 1
0 2 0 0
2 3 6 4
n n n n
 xi  2
i 1
 yi  6
i 1
 xi yi  5
i 1

i
1
xi2  14

Applying theorem 3.2.1, we have

n n n
n xi yi  xi  yi
4  5    2  6 
a  i 1 i 1 i 1

4 14    2 
2 2
n  n 
n xi2    xi 
i 1  i 1 

8
 and
13

1  n n  1  16 
b  y  a  xi   6 
n  i 1 i i 1  4  13 
47

26

Thus the least squares regression line is

8 47
y x
13 26

51
which is shown in the figure below

8 47
f  x  x
13 26
y

(2,3)

2
(0,2)

(-1,1)

x
(-3,0)
Fig 3e

3.3. Lagrange Multipliers

In this subsection we describe a general method of finding the extreme values of a function
subject to a constraint.

3.3.1. The Lagrange Method for Functions of Two Variables

Let us consider the problem of finding an extreme value of a function f of two variables subject
to a constraint of the form g  x, y   C , that is, we seek an extreme value of f on the level curve
g  x, y   C (rather than on the entire domain of f). If f has an extreme value on the level curve
 
at the point x0 , y0 , then under certain conditions there exists a number  such that

 
grad f x0 , y0   grad g x0 , y0   __________ 1
where
   
grad f x0 , y0  i f x x0 , y0  j f y x0 , y0  
  
 f x x0 , y0 i  f y x0 , y0 j 

Consider the figure below, which shows the level curve C of g along with several level curves of
f, and assume that


grad f x0 , y0  0  .

52

grad g x0 , y0  y

C
f  x, y   6
g  x, y   c
f  x, y   5

f  x, y   4

f  x, y   3 x
0
f  x, y   2 Fig 3f


Let x0 , y0  be the point in the figure above at which f assumes its maximum value 6 on C.
Observe that C cannot cross the level curve f  x, y   6 because if it did, C would intersect level
curves of f corresponding to values of f larger than 6, and that would contradict the assumption
that 6 is the maximum value of f on C. Therefore C and the level curve f  x, y   6 have the
 
same tangent at x0 , y0 . Consequently grad g x0 , y0   which is normal to C at  x0 , y0  is
 
parallel to grad f x0 , y0 which is normal to the level curve f  x, y   6 at x0 , y0 .  
Theorem 3.2.2 (Lagrange’s Theorem)

Let f  x, y  and g  x, y  have continuous first partial derivatives, and suppose f has an
 
extremum f x0 , y0 when  x, y  is subject to the constraint g  x, y   0. If V g x0 , y0  0 ,  
   
then there is a real number  such that V f x0 , y0   V g x0 , y0 where V denotes gradient
vector.

Proof

The graph of g  x, y   0 is a curve C in the xy- plane. Under certain conditions we can represent
C in parametric form of

x  k t  , y  h t  ,
where t is in some interval I.

Let r  t   x i  y j  k  t  i  h t  j
be the position vector of point p  x, y  on C (see figure 3g below).

53
 
y
r  t0

g x0 , y0 

p0 x0 , y0 
 
r t0
p  x, y 
r t 
c : g  x, y   0

x
Fig 3g
 
and let the point p0 x0 , y0 correspond to t  t0 ; that is

 
r t0  x0 i  y0 j  k t0 i  h t0 j    
Now we define a function F of one variable t by

F t   f  k t  , h t 

such that as t varies we obtain functional values f  x, y  that correspond to  x, y  on C; that


 
is f  x, y  is subject to the constraint g  x, y   0 . Notice that f x0 , y0 is an extremum of f
under these conditions and it therefore follows that

      
F t0  f k t0 , h t0

is an extremum of F  t  . Thus F  t0  0 .  
Since F is a composite function, then by the Chain Rule

dx dy
F   t   f x  x, y   f y  x, y 
dt dt

and therefore, at the extremum,

 
0  F  t0

    
 f x x0 , y0 k  t0  f y x0 , y0 h t0   

= V f x0 , y0  r  t0   
(by dot product of two vectors).

54
   
This shows that the vector V f x0 , y0 is orthogonal to the tangent vector r  t0 to C.
   
However, V g x0 , y0 is also orthogonal to r  t0 , because C is a level curve for g. Since
V f  x0 , y0  and V g  x0 , y0  are orthogonal to the same vector they are parallel, that is

 
V f x0 , y0   V g x0 , y0  
for some  . The number  is called a Lagrange multiplier.

The method of Lagrange multipliers uses theorem 3.3.2 to find the extreme values of a function f
subject to a constant.

3.3.3 Method of Lagrange Multipliers

Let f and g satisfy Lagrange’s theorem, and let f have an extremum subject to the constraint
g  x, y   C. To find the extremum of f, use the following steps.

1. Simultaneously solve the equations

V f  x, y    Vg  x, y  and g  x, y   C by solving the following system of equations

f x  x, y    g x  x , y 
f y  x, y    g y  x , y 
g  x, y   C

2. Evaluate f at each solution point. The largest value yields the maximum of f subject to the
constraint g  x, y   C , and the smallest value yields the minimum of f subject to the
constraint g  x, y   C .

Example 3.3.4

Find the maximum value of

f  x, y   4 xy, x  0, y  0

subject to the constraint

x2 y 2
 1
9 16

55
Solution

In this case,
x2 y 2
f  x, y   4 xy and g  x, y    1
9 16
By Lagrange’s theorem
V f  x, y    Vg  x, y  and
g  x, y   0.
Hence
 2x   2y 
4 y i  4x i     i     i
 9   16 
x2 y2
  1  0.
9 16

2x y
4y  , 4x 
9 8
2 y 2
x
  1  0.
9 16

18y
  and substituting in the second equation above we obtain
x
y  9y 
4x   
4 x 
 16 x 2  9 y 2 .
9 2
 x2  y .
16

and substituting this value in the third equation we obtain


1 9 2 1 2
 y   y 1
9  16  16
 y2  8
y  2 2.

But y  0 by hypotheses, hence y  2 2 .

 
9 2
When y  2 2 ; x 2  2 2
16
9 9
 x2  x 8
16 2

56
Hence
3
x .
2

Thus the maximum of f is

 3   3 
f
 2
, 2 2   4
  2 

 2 2  24
Example 3.3.5

Find the extrema of f  x, y   xy if  x, y  is restricted to the ellipse 4 x2  y 2  4.

Solution

By Lagrange’s theorem we let f  x, y   xy and g  x, y   4 x 2  y 2  4


Then we solve
V f  x, y    Vg  x, y  and
g  x, y   0

y i  x i  8x  i  2 y  i
4 x2  y 2  4  0 __________ 1

y  8 x
x  2 y
4 x2  y 2  4  0 _________ 2
From first equation above we have that
y

8x

and substituting in the second equation we obtain


2
 y  y
x  2y   
 8x  4 x
 4x2  y 2
and substituting in the third equation we obtain

y2  y2  4  0  2 y2  4
y 2

57
 
2 2
when y   2 , 4x   2

 x2 
1
2
1 2
x 
2 2

Also solving the first two equations of equation 2 simultaneously yields


x  16 x   0, or x 1  16 x    0
2 2
 
which gives
1
either x  0 or    . When x  0 , then using 4 x2  y 2  4  0 . We obtain y  2. Thus
4
 2   2   2 
 2 , 2  ,  2 ,  2  ,   2 ,  2  .
possibilities of extrema are (0, -2), (0,2), 
     

The value of f at each of the points is given in the table below:


(x, y) (0,2) (0, -2)  2   2   2   2 
 , 2   ,  2    , 2    ,  2 
 2   2   2   2 
f  x, y 0 0 1 -1 -1 1

 2   2 
It follows that f  x , y  takes on a maximum value 1 at either  , 2  or   ,  2  and a
 2   2
   
 2   2 
minimum value -1 at 
 2 ,  2 

or  
 2 , 2  .
   
Lagrange’s theorem can be extended to functions of more than two variables. For example, to
determine the extrema of f  x , y, z  subject to the constraint g  x , y, z   0 . We begin by
finding the (simultaneous) solutions of the equations

V f  x, y, z    Vg  x, y, z  and
g  x, y, z   0

Example 3.3.6

Find the minimum value of


f  x, y, z   2 x2  y 2  3z 2
subject to the constraint
2 x  3 y  4 z  49
Solution

58
Let g  x, y, z   2 x  3 y  4 z  49. Then

V f  x, y, z   4 xi  2 yj  6 zk
and
V g  x, y, z   2i  3 j  4k
 V g  x, y, z   2 i  3 j  4 k

Then by Lagrange’s theorem

4 x  2
2 y  3
6 z  4
2 x  3 y  4 z  49

From the first three equations we have

 3 2
x , y and z 
2 2 3
and substituting in the fourth equations;

   3   2 
2    3     4     49
 2  2   3 
9 8
      49
2 3
 6
Hence x = 3, y = -9 and z = -4.

Therefore the minimum value of f is f  3, 9, 4   2  3   9   3  4   147


2 2 2

Example 3.3.7

If f  x, y, z   4 x 2  y 2  5z 2 , find the point on the plane 2 x  3 y  4 z  12 at which f  x, y, z 


has its least value.

Solution

We wish to find the minimum of f  x, y, z   4 x2  y 2  5z 2 subject to the constraint


g  x, y, z   2 x  3 y  4 z  0.

59
V f  x, y, z   8 xi  2 yj  10 zk
 V g  x, y, z   2 i  3 j  4 k

By Lagrange’s theorem

8 x  2
2 y  3
10 z  4
2 x  3 y  4 z  0.

From the first three equations, we have

2 5
  4x  y  z.
3 2
These conditions imply that

8
y  6 x and z  x.
5

Substituting into the equation

2 x  3 y  4 z  12  0, we obtain

32
2 x  18 x  x  12  0
5

5 30 8
which yields x  . Hence y  and z  . Thus the minimum value occurs at the point
11 11 11
 5 30 8 
 , , .
 11 11 11 

Remark

For some applications there may be more than one constraint. In the next subsection we consider
for the case of two constraints.

3.3.8 Lagrange Multipliers with two constraints

Consider the problem of finding the extrema of f  x, y, z  subject to the two constraints

g  x, y, z   0 and h  x, y, z   0

60
It can be shown that if f has an extremum subject to these constraints, then

V f  x, y, z    V g  x, y, z    V h  x, y, z 

where  is a second Lagrange Multiplier.

Example 3.3.9

Let
T  x, y, z   20  2 x  2 y  z 2

represent the temperature at each point on the sphere x2  y 2  z 2  11. Find the extreme
temperatures on the curve formed by the intersection of the plane x  y  z  3 and the sphere.

Solution

In this case we have two constraints

g  x, y, z   x 2  y 2  z 2  11
h  x, y , z   x  y  z  3

Since

V T  x, y, z   2i  2 j  2 zk ,
 V g  x, y, z   2 xi  2 yj  2 zk

 V h  x, y, z    i   j   k , we have the following system of equations

2  2 x  
2  2 y  
2 z  2 z  
x 2  y 2  z 2  11
x yz 3

By subtracting the second equation from the first, we obtain the following system
  x  y  0
2 z 1       0
x 2  y 2  z 2  11
x yz 3

61
From the first equation we conclude   0 or x  y . We consider these two cases separately. If
  0, then since 2  2 x   , we have   2, and, by substituting into the second equation we
have z  1. The last two equations now become

x 2  y 2  10
x y 2

and, by substitution, we obtain

x2   2  x   10  0
2

which simplifies to

x2  2 x  3  0

with solutions x  3 and x  1 .

The corresponding y-values are y = -1 and y = 3, respectively. Thus if   0 the critical points are
(3, -1, 1) and (-1, 3, 1). If   0 , then x = y, and the last two equations become

2 x 2  z 2  11
2x  z  3

and, by substitution, we obtain 2 x2   3  2 x   11  3x2  6 x  1  0.


2

The solutions are x 


3 2 2
, and the corresponding values of z are x 

3 4 3 , which
3 3
yield two additional critical points. Finally to find the optimal solutions, we compare
temperatures at the following critical points.

T  3,  1, 1  T  1, 3, 1  25
 3  2 3 3  2 3 3  4 3  91
T   , ,   30.33
 3 3 3  3
 3  2 3 3  2 3 3  4 3  91
T   , ,  30.33
 3 3 3  3

91
thus, T = 25 is the minimum temperature and T  is the maximum temperature on the curve.
3

62
3.4 Self Test 3
1

1. Determine the relative extreme of f  x, y   1  x  y 2 3
2

2. Examine the following functions for relative extrema and saddle points

(a) f  x, y   5x2  4 xy  y 2  61x  10 (b) h  x, y   x2  y 2  2 x  4 y  4


(c) f  x, y   x3  3xy  y3

3. A retail outlet sells two competitive products, the prices of which are p1 and p2. Find p1 and
p2 so as to maximize total revenue, where
R  500 p1  800 p2  1.5 p1 p2  1.5 p12  p 22

In questions 4-5, find the least squares regression line for the given points

4. (0, 0) , (1,1), (3, 4), (4, 2), (5, 5) 5. (0, 6), (4, 3), (5, 0), (8, -4), (10, -5)

In exercises 6 – 15 use Lagrange multipliers to find the local extrema of the function f subject to
the stated constraints

f  x, y   xy
6. Maximize
Constraint : x  y  10

f  x, y   x 2  y 2
7. Minimize
Constraint : x  y  4  0

8. f  x, y   y 2  4 xy  4 x 2
2 2
Constraint x  y  1

f  x, y, z   x2  y 2  z 2
9.
Constraint x  y  z  1

10. f  x, y, z   x2  y 2  z 2
Constraint x  y  z  25

11. Maximize f  x, y   x 2  y 2
Constraint y  x2  0

63
12. Minimize f  x, y   x 2  y 2
Constraint 2 x  4 y  15  0

13. Maximize f  x, y   e xy
Constraint x2  y 2  8  0

14. Minimize: f  x, y, z   x2  y 2  z 2
Constraint: x  y  z  6  0

15. Maximize f  x, y   xyz


Constraint x  y  z  6  0
In 16 – 20, use Lagrange multipliers to find the indicated extrema of f subject to two constraints.
In each case, assume x, y and z are positive.

16. Maximize f  x, y, z   xyz


Constraints
x  y  z  32
x yz 0

17. Maximize f  x, y, z   xy  yz
Constraints
x  2y  6
x  3z  0

18. f  x, y, z   x2  y 2  z 2
Constraints
x  y 1
y2  z2  1

19. f  x, y, z   z  x 2  y 2
Constraints
x  y  z 1
x2  y 2  4

20. Minimize f  x, y, z   x2  y 2  z 2
Constraints
x  2z  4
x y 8

64
3.5 Solutions to Self Test

1. (0, 0) is a relative maximum of 7.

2. (a) Relative maximum: (8, 16, 74)


(b) Saddle point: (1, -2,-1)
(c) Saddle point: (0,0,0)
Relative minimum: (1, 1, -1)

3. p1  $586.67 , p2  $840.00
37 7
4. y  x
43 43
175 945
5. y   x
148 148
6. f  5,5  25
7. f  2, 2   8

 2 1   2 1 
8. Extrema occur at  ,  and  , 
 5 5  5 5

1 1 1 1
9. f  , , 
3 3 3 3

f  x, y , z   x 2  y 2  z 2
10.
g  x, y, z   x  y  z  25  0
f x  2 x, f y  2 y, f z  2 z, g x  1, g y  1, g z  1
2 xi 2 yj  2 zk   i   j   k
x  y  z  25  0
2 x   , 2 y   , 2 z  
x  y  z  25  0
  
   25  0
2 2 2
 x, y, z   
25 25 25 
, , 
 3 3 3 
 25 25 25  625 625
f  , ,  x3 
 3 3 3  9 3

 2 1 1
11. f  ,  
 2 2 4

65
3  3
12. f  , 3   5
2  2

13. f  2, 2   e4

14. f  2, 2, 2   12

15.

16. f 8,16,8  1024

 3 
17. f  3, ,1  6
 2 

18. f  0, 1, 0   1 is minimum


f  2, 1, 0   5 is maximum

19. Minimum: f  2, 2, 1  2 2   2 2  3
Maximum: f   2,  2, 1  2 2   2 2  3

20.

66
LECTURE 4

INTEGRAL CALCULUS OF FUNCTIONS OF SEVERAL VARIABLES

4.0 Introduction

In this section we extend the knowledge about integration of functions of single variables to
b
integration of functions of two and three variables. The definite integral  f  x  dx is defined
a
with respect to a function f  x  defined over an interval a  x  b (see figure 4a). The double
integral

D  f  x, y  dx dy
will be defined with reference to a function f  x, y  defined over a closed region D of the xy
plane (see figure 4b).

f(x)
f(x,y)

V
y
a b

Fig 4a: Area D


Fig 4b: Volume
x

Objectives

At the end of this lecture you should be able to:

1 Define double Riemann Sum


2 Define and apply integrals in polar coordinates
3 Evaluate surface areas of various solids
4 Define and evaluate triple integrals
5 Determine volumes of solid regions.

67
4.1. Review

Recall that an area function for f  x  is a function B  u, v  which has the following three
properties:

1. Addition Property

B  u, v   B  v, w  B  u, w
when u  v  w.

2. Rectangle Property

m  v  u   B  u, v   M  v  u 
where m is the minimum and M the maximum value of f  x  on u, v .

B  u, u   0, B  v, u   B u, v 
3.
For functions of two variables, instead of closed intervals u, v  in the line, we deal with
closed regions D in the plane. The analogue of an area functions is a volume function. We
denote V(D) to be volume of the solid over the region D between the (x,y) plane and the
surface z  f  x, y  .

A volume function for f  x, y  is a function B which assigns a real number B(D) to each
closed region D and has the following two properties:

1. Addition Property

If D is divided into two regions D1 and D2 which meet only on a common boundary
curve, then
   
B  D   B D1  B D2
Intuitively, the volume over D is the sum of the volumes over D1 and D2 .

2. Cylinder Property

Let m and M be the minimum and maximum values of f  x, y  on D and let A be the area
of D. Then

mA  B  D   MA
intuitively, the volume over D is between the volumes of the cylinders over D of height m
and M. This corresponds to the Rectangle property for single integrals.

68
Definition 4.1.1

A closed region in the  x, y  plane is a set D of real points  x, y  given by inequalities


a1  x  a2 , b1  x   y  b2  x 

where b1  x  and b2  x  are continuous and b1  x   b2  x  for x in  a1 , a2  . (see figure 4c). The
simplest type of a closed region is a closed rectangle
a  x  a2 , b1  y  b2

Shown in figure 4d.

y y

b2(x) b2

D
b1
b1(x)

a1 b1 x a1 b1 x
Fig 4c: closed region Fig 4d: closed rectangle

Definition 4.1.2

The boundary of D is the set of points in D which are on the curves

x  a1, x  a2 , y  b1  x  , y  b2  x  .

An open region is a set of real points defined by strict inequalities of the form
c1  x  c2 , d1  x   y  d2  x 

Remark

Whenever we refer to a function f  x, y  and a region D, we assume that f  x, y  is continuous


on some open region containing D.

If f  x, y   0 on D, the double integral is intuitively the volume of the solid over D between the
surfaces z  0 and z  f  x, y  ; i.e. the solid consisting of all points  x, y, z  where  x, y  is in
D and 0  z  f  x, y  .

69
If f  x, y   0 on D the double integral is intuitively the negative of the volume of the solid
under D between the surface z  f  x, y  and z  0 . Thus the volumes above the plane z  0 are
counted positively and volumes below z  0 are counted negatively.

4.2. Double Riemann Sum

We can now define the Double Riemann Sum and use it to give a precise definition of the double
integral. First consider the case where D is a rectangle
a1  x  a2 , b1  y  b2
shown in figure 4d.

Let x and y be positive real numbers. Partition the interval  a1 , a2  into subintervals of
length x and b1 , b2  into sub intervals of length y . The partition points are:
x0  a1, x1  a1  x, x2  a1  2x,..., xn  a1  nx
y0  b1, y1  b1  y, y2  b1  2y,..., y p  b1  py

where
xn  a2  xn  x, y p  b2  y p  y

We have partitioned the rectangle D into x by y sub rectangles with partition points
 xk , yl  , 0  k  n, 0  l  p ,
as shown in figure 4e

b2
yp
yp-1
y

y2
y1
y0 b1
a1 x
x
x0 x1 x2 xn-1 xn a2
Fig 4e

70
Definition 4.2.1

The Double Riemann Sum for a rectangle D is the sum

n p
 D  f  x, y  x y    f  xk , yl  x y
k 0 l 0


This is the sum of the volume of the rectangular solids with base x y and height f xk , yl .
The formula

 D  f  x, y  x y
approximates the volume of the solid D between z  0 and z  f  x, y  . Consider a general
region D given by a1  x  a2 , b1  x   y  b2  x  .

The circumscribed rectangle of D is the rectangle a1  x  a2 , B1  y  B2 where

B1  minimum value of b1  x 
B2  maximum value of b2  x 

as shown in figure 4f
y

b2
b2(x)

The circumscribed rectangle


b1
b1(x)

a1 a2 x
Fig 4f

Given positive real numbers x and y , we partition the circumscribed rectangle of D into
 
x by y sub rectangles with partition points xk , yl , 0  k  n, 0  l  p.

Definition 4.2.2

The Double Riemann Sum over D is defined as the sum of the volumes of the rectangular solids
 
with base x y and height f xk , yl which belong to D. In symbols

71
 D  f  x, y  x y    f  xk , yl  x y
 xk , yl  inD
Remark

Notice that in the Double Riemann Sum over D, we only use partition points (x,y) which belong
to D. Such points are shown marked in figure 4g.
y

* *
Partition points that belong to D
* * * * *
D
* * * * *

x
Fig 4g

We give the following examples to illustrate the Double Riemann Sum.

Example 4.2.3

Find the Double Riemann Sum


 D1  x2 y x y
1 1
where D1 is the square 0  x  1, 0  y  1 and x  , y  .
4 5
Solution

The partition of D1 is shown in figure 4h and the values of x 2 y at the partition points are shown in
the table below.
y
1
4
5* * * *
3
* * * * D1
5
2
* * * *
5
1
* * * *
5
* x
0* 1
*2 *3
1
4 4 4

72
f  x, y   x 2 y y0  0
y1 
1
y2 
2
y3 
3
y4 
4
5 5 5 5
x0  0 0 0 0 0 0
1 0 1 2 3 4
x1 
4 80 80 80 80
2 0 4 8 12 16
x2 
4 80 80 80 80
3 0 9 18 27 36
x3 
4 80 80 80 80
1 1 1
 D1  x 2 y x y  1  2  3  4  4  8  12  16  9  18  27  36  x x x
80 4 5
 0.0875

1 1
Use similar computation with x  , y  to show that
10 10
 D1  x2 y x y 0.12825

Example 4.2.4

Find the double Riemann Sum


 D  x2 y x y
2
where D 2 is the region
1 1
0  x  1, x2  y  x and x  , y  .
4 5

Solution

The circumscribed rectangle of D 2 is the unit square. The partition and D 2 are shown in figure
4I and the partition points which actually belong to D 2 are circled.
y x
y
1
4
5 *
3
* *
5
2 y  x2
* * Fig 4I
5
1
*
5
x
0* 1 2 3 1
4 4 4 73
f  x, y   x 2 y y0  0
y1 
1
y2 
2
y3 
3
y4 
4
5 5 5 5
x0  0 0
1 1 2
x1 
4 80 80
2 8 12
x2 
4 80 80
3 27 36
x3 
4 80 80

The Double Riemann Sum is

 1 2 8 12 27 36  1  1 
 D1  x2 y x y   80  80  80  80  80  80  4  5 
86
  0.05375
80 x 20

1 1
Use similar computation with x  and y  to
10 10

show that

 D2  x2 y x y 0.04881.

Remark

Given the function f  x, y  and the region D, the Double Riemann Sum
 D  f  x, y  x y
is a real function of x and y . When we replace x and y by positive infinitesimals dx and
dy, we obtain the infinite Double Riemann Sum

 D  f  x, y  dx dy .

74
4.2.5 Self Test

Compute the following Double Riemann Sums

1 1
1.  D   3x  4 y  x y; x  4 , y  4
D :0  x  1, 0  y  1

1 1
2.  D   4  2 x  5 y  x y; x  2 , y  5
D :  2  x  2,  1  y  1

 
 D   cos x sin y  x y; x  6 , y  6
3.
 
D:  x , 0 y 
2 2

 1
4.
 D  y sin x x y; x  4 , y  4
D :0  x   , sin 2 x  y  2sin x

4.26 Solutions to Self Test

1. 2.625

2. 6

 7  4 3  36
2
3. 3.82

4. 7 2  22  B4

75
4.3 ITERATED INTEGRALS

A double integral can be evaluated by two single integrations. First let us define vertically and
horizontally simple regions.

Definition 4.3.1

(a) A plane region R is vertically simple if there are two continuous functions g1 and g2 on an
interval  a, b such that g1  x   g2  x  for a  x  b and such that R is the region between
g and g2  a, b
the graphs of 1 on , see figure 4k. In this case we say that R is the vertically
simple region between the graphs of g1 and g2 on  a, b .

(b) A plane region R is horizontally simple if there are two continuous functions h1 and h2 on an
interval  c, d  such that h1  y   h2  y  for c  y  d and such that R is the region between
the graphs of h1 and h2 on  c, d  , see figure 4l. In this case say that R is the horizontally
simple region between the graphs of h1 and h2 on  c, d  .

(c) A plane region R is simple if it is both vertically simple and horizontally simple (see figure
4m).
y
y y
g2
d
g2 d
h1 h2 h1 h2

c
g1 g1
c
x x
a b x a b
Fig 4k Fig 4l Fig 4m
Vertically simple Horizontally simple Simple region
region region

Let f be non negative and continuous on a vertically simple region R between the graphs of
g1 and g2 on  a, b . Let D be the solid region between the graphs of f and R. Since f is
continuous in each variable separately, the cross sectional area A  x  of D at any x in  a, b is
given by

76
g2  x 
A x   f  x, y  dy.
g1 x 

it can also be shown that A is continuous on  a, b ; hence the volume V of D is given by

b
V   A  x  dx 

b  g2 x 
f  x, y  dy  dx
a   

a  g1 x 
 

But V is also given by

V   f  x, y  dA
R

where dA  dx dy or dA  dy dx.

It turns out that the two formulas for V yield the same result. Therefore

b  g2  x  
  f  x, y  dA   a   g1 x  f  x, y  dy  dx
R  

Similarly, if R is the horizontally simple region between the graphs of h1 and h2 on  c, d  , then

f  x, y  dA 
  f  x, y  dx  dy
d  h2 y
   
c  h1 y
R  
The integrals


b  g2 x 
f  x, y  dy  dx and
d  h2 y  
f  x, y  dx  dy
   
a  g1 x 
c  h1 y   
   

are called iterated integrals, because they are performed iteratively. Normally we write them as

b g2  x  b  g2  x  
 a  g1 x  f  x, y  dx dy for  a   g1 x  f  x, y  dy  dx
 

and
d   f  x, y  dx dy for d  h2 y  f  x, y  dx  dy
h2 y
  h1 y 
c  c  h1 y 
 

Furthermore if R is a simple regression then

77
b g2  x  d h2  y 
  f  x, y  dA  a g1 x  f  x, y  dydx  c h1 y  f  x, y  dxdy
R

Example 4.3.2

Let R be the rectangular region bounded by the lines x  1, x  2, y  0 and y  2. Find

x
2 y dA
R

Solution

The region R is a simple region shown in figure 4n below

Leaves at y = 2

x
-1 2
Enters y = 0

First regarding R a vertically simple region, draw a vertical line through R. The point where it
enters R, y  0 gives the lower boundary of integration and the point where it leaves R, y  2
gives the upper boundary of integration. Therefore

2
2 2 2  y2 
 
x y dy  x 2 2
y dy  x  
0 0

2  0
 2 x2
Then
2 2 2 2
  f  x, y  dA 1 0 x y dy dx 
1 
2 x 2 dx
R
2
2 
  x3   6
 3  1

secondly, regard R as the horizontally simple region between the graphs of x  1 and x  2 for

78
0  y  2. Trace a horizontal ray through the region R as shown in figure 4p.
y

Leaves at y = 2

Enters x = -1
x
-1 0 1 2
Fig 4p

Therefore,

2 2 2 2  2 
 f  x, y  dA  
x y dx dy  y  x 2 dx  dy  
0 1 0  1 
R
2
2  x3  2
0 
 y 
3   1
dy  0 3 y dy
2
 3 y2 
  6
 2  0

Example 4.3.3

Let R be the triangular region between the graphs of y  1  x and x  axis on  2, 1 , and
suppose that f  x, y   4  y in R.

Evaluate

  f  x, y  dA
R

79
Solution

The region R shown in the figure below is simple.


y

y=1-x

x
-2 -1 0 1

Fig 4q

Regarding R as the vertically simple region between the graphs of


y  0 and y  1  x for - 2  x  1

It follows that

1 x
1 1 x 1  y2 
 f  x, y  dA 
   4  y  dy dx  2 4 y  
 dx
2 0
R 2  0
1  1 x 
2
 4 1  x    dx
 
2  2 
 
1 7 x 
2
    3x   dx
2 2
 2 
1
 7x 3 x3 
   x2  
 2 2 6 
2
27

2

If R is regarded as the horizontally simple region between the graphs of


x  2 and x  1  y for 0  y  3

80
We find that

3 1 y
  f  x, y  dA 0 2  4  y  dx dy
R
3 1 y
   4  y  x 2 dy
0
3
  4  y  1  y    2  dy
0

3

0 
12  7 y  y 2 dy
3
 7 2 y3 
 12 y  y 
 
 2 3 
 0
27

2

Remark

We observe that when a region R is simple, then we obtain the same result whether we consider
R as a vertically simple region or a horizontally simple region.

Example 4.3.4

Find the volume V of the solid region D bounded above by the paraboloid z  4  x2  y 2 and
below by the xy-plane.
z
4
4
z  4  x2
D

-1
-1 0 R
-2 2

1 y  4  x2

x Fig 4r

81
The intersection of D and the xy-plane is the region R bounded by the circle x2  y 2  4 . By
definition

  4  x 
V 2  y 2 dA
R

Since R is the vertically simple region between the graphs of

y   4  x2 and y  4  x 2

for 2  x  2 we conclude that

V 
2 4 x2

2  4 x2  4  x2  y2 dy dx
4 x 2
 3
2
2



   4  x2 y 
y

3 
 2
dx
 4 x
 3
2  4  x  2
 
2

 2  4  x 2 4  x2   dx
2 3
 
 
3
4 2
  4  x 2 2 dx
3 2
 
Let x  2sin 
x 
-2 
2
2 
2
dx  2cos  d

Therefore
3
4 2
V   4  x 2 2 dx
3 2
 
 3
4

 2 4  4 cos2  2 2 cos  d
3

2

82

64 2

3 
cos4  d
2

64  1 3 3  2
  cos3  sin   cos  sin   
3 4 8 8  
2
by reduction formula
 8

Example 4.3.5
5
Let D be the region bounded by the curve xy  1 and the line y   x.
2

Find the inequalities which describe D, and write down an iterated integral equal to
  f  x, y  dA
R

Solution

Sketch the region D as shown in the figure 4s below


y
5
y x
2

D
xy = 1

The line and curve intersect where

5  1
x    x   1  x  and x  2
2  2

1 1 5
For  x  2, the curve y  is below the line y   x. Therefore D is the region
2 x 2
1 1 5
 x  2,  y  - x
2 x 2

83
The inequalities for x give the limits of the outside integral and those for y give the limits of the
inside integral. Thus

5
2 2x
 f  x, y  dA  1 1
2 f  x, y  dy dx
R x

Example 4.3.6

Evaluate the double integral

 

0 2 y 2 y
2 sin x 2 dx dy

Solution

Limits of integration are


 
0 y and y  x 
2 2
 
y  0, y  , y  x and x 
2 2

the region R under integration is shown in figure 4t


y

 y= x
2

Leaves at y = x R

x

Enters at y = 0
2

84
Consider the region as a vertically simple region and integrate as

 
x 2 x
0 2
 y sin x dy dx  
2 2sin x 2   y 2dy  dx
0 0  0 
 3 x 
2y 1
  sin x   dx   2 x3 sin x 2dx
2
0 3 0
 3  0
Let
x u
0 0
 
2 2
u  x2
du
 dx
dx

Therefore

1 2 3 1 2 3 du
 x sin x dx   x sin 4 
2
3 0 3 0 dx

1
  2 u sin u du
6 0
   
1 
   u cos u  02   2 cos u du 

6 0 

by applying integration by parts.

   
1 
   u cos u  02   sin u  02 
6
 

1        
   cos  0  sin  sin 0 
6 2 2   2  
1
 .
6

One order of integration will often produce a simpler integration problem than the other order.
This is evident in the previous example 4.3.6. In such cases, you would be requird to change the
order of the integral.

85
Example 4.3.7

By reversing the order of integration, evaluate

9 3
0  y
sin  x3dx dy

Solution

To reverse the order of integration we must determine the region R over which the integration is
performed.
y

y = x2
x=3

x
3
y=0
Fig 4u

From the limits on the given iterated integral we infer that R is the horizontally simple region
between the graphs of x  y and x  3 for 0  y  9
(see figure 4u above).

Since R is also the vertically simple region between the graphs of

y  0 and y  x2 for 0  x  3
We find that

9 3
0  sin  x3dx dy    sin  x d A
3
y
R
3 x2
  
0 0
sin  x3dy dx

86
3 x2
  y sin  x3  y 0 dx
0
3
3  1 
  x 2 sin  x3 dx   cos  x3 
0  3 0
2

3
Example 4.3.8

Let R be the region between the graphs of y  x and y  x3 .

Evaluate    x  1 d A.
R

Solution

The graphs of the two equations intersect at (x, y) if x  x3 which happens if x = -1, x=0
or x = 1. Therefore R is composed of the two vertically simple regions R1 and R2 as depicted in
figure 4v.
y

y=x
1

R2
y = x3

0 x
-1 1

y = x3
R1

Fig 4v

87
Consequently

   x  1 d A     x  1 d A     x  1 d A
R R1 R2
x3
0 1 x

1 x
x  1 dy dx 
0 x3
 x  1 dA

0 x3 1 x

1 x  1 y x dx   x  1 y x3 dx
0 
 x  1  x    dx
0 1
 3  x dx    x  1 x  x
3
1 0


0
1  x4  x3  x2  x  dx    x4  x3  x 2  x  dx
0
1
0 1
x5 x 4 x3 x 2 x5 x 4 x3 x 2
       
5 4 3 2 5 4 3 2
1 0
1

2

88
4.3.9 Self test

In 1 – 4 evaluate the double integrals

1.    3x  4 y  d A , D :0  x  1, 0  y  1
D

 x 
2. 2  y 2 d A , D :  2  x  2,  2  y  2
D

x
3.   y d A, D :1  x  2, 1  y  2
D

2 x y
4.  e d A , D :  2  x  2,  2  y  2
D

In 5-13 evaluate the iterated integral and show that it is equal to the iterated integral in reversed
order.
1 2
0  0 dy dx 1 1 y
2
5. 6. 0   1  y 2 dx dy

1 3 y

0 y2
dx dy 2 1
7. 8. 0  x dy dx
2

0  0  x 
1 1 1 6
0 1
9. 2 y  3xy 2  5 dy dx 10. x  y dy dx


2 1 cos y x
11.  0 0
r 1  r 2 dr d 12.  
4
0 0
e sin y dx dy

1
1
10  0 y e
y xy
13. dx dy

In 14 – 19, evaluate the given iterated integral.

1  0  
2 4 2 2 y  y2
14. x 2  2 y 2  1 dx dy 15. 0  3 y 2 6 y
3 y dx dy

89

1 1 y2 2 cos 
16. 0  0  x  y  dx dy 17.  
2
0 0
r dr d
 
sin  cos  2
18.  
2
0 0
 r dr d 19.  
4
0 0
3r sin  dr d

In 20 – 26 reverse the order of integration and evaluate the resulting integral

2 5  x12 dx dy 1 1 x2
20. 0 1 y 2 ye 21. 0 y e dx dy

1 2 3
3
3 1 1
  0 y sin x
2
22. 3 2 sin x 2 dx dy 23. dx dy
0 y

2 2  y2 2 4
24.  
0 x
e dy dx 25. 0 y2 x sin x dx dy

2 2
26. 0 x x 1  y3 dy dx

Solutions to Self Test 4.3.9

1
1 1  1  3x 2
13 
1.    3x  4 y  dx dy     4 xy  dy     4 y  dy
0 x 0 2
  0 0 2 
1
3 2 3 5
  y  y   1 
2 0 2 2

2
 
 
3
2 2 x 8   8 
2.   x 2  y 2 dx dy    xy 2     2 y2      y2 
2 8  3 
  2  3   3 
2
 16 2  16 3  32   32 
    3 y  dy   y  y     8     8
 3   3  2  3   3 
64 64  48 112
  16  
3 3 3

90
2 2
x 2 2 2 2
 x  dy  dy  3  ln y 2
3. 1 1 y dx dy   1  1 1 2 1

21 2 2
 x 2   3 ln y  3 ln 2
1 y  1 2 1 2
3
 ln 2
2

e y  2 x  2
2 2 e y 4

4 4
4  e  e   y  2
4. 
 2 2 2 
e  e e  e  2
2 2   2
4 4 2
e4  e4  2 2  e  e e  e
e e 
2
  
2   2

1 1
0  y 0 dx 20 dx 2  x0  2
2 1
5.

2
1 1 y 1 
6. 0  x   1 y
2
dy   2 1  y 2 
0 2

1
1  1 3
x
2 3 4 3 5 
7.   x  2    y 3  y 3  dy   y 3  y 3 
0  y   4 5 
   0
3 3 15  12 5
   
4 5 20 12

2 x
2
2  1  x2 
8.  2 

y  1   dx   x    2  1  1
0   2 0  4 0
 
1

1
1 2
y2  1  x2  1  x3 x 2 
9.   xy  5 y      x  5 dx      5
3
0
x
0 2 0 6
 2 0    2 
1 1 1  3  30 28 14
 5   
6 2 66 6 3

10.

91
2 2
2
11. 12. e  e 2  1
3 2

1 0  x 
2 4 20
13. 9 1  e  14. 2
 2 y 2  1 dx dy 
3

2
15. 40 16.
3

 2 1
17. 18. 
2 32 8

19.
3 
32 
2  2  20.
1 16
4

e 1 
1 4
21.  e  1 22.
4 3

1
23. 1  cos1 0.230 24. x  y 2 , x  4, y  0, y  2
2

26
25. - 4 cos4 – sin 4 26.
9

92
4.4 Double Integrals in Polar Coordinates

Some double integrals are much easier to evaluate in polar form than in rectangular form. This
happens in regions such as cardioids, circles, lemniscates and for integrals involving the sum
x 2  y 2 . The type of region over which we will integrate in polar coordinates can be described
as follows. Suppose that h1 and h 2 are continuous on an interval  ,   , where

0      2 , and that 0  h1    h2   for     .

Let R be the region in the xy plane bounded by the lines    and    and by the polar
graphs of r  h1   and r  h2  
y
r  h2  

 

   

r  h1  

Fig 4 f: The region between the polar graphs of h1 and h2 on  ,   .

We say that R is the region between the polar graphs of h1 and h2 on  ,   . Such a region R
need not be vertically or horizontally simple. It turns out that every function that is continuous
on R is integrable on R, so
  f  x, y  d A exists
R

We state the following theorem which gives a summary of the results of this subsection.

Theorem 4.4.1

Suppose that h1 and h2 are continuous on  ,   , where 0      2 , and that


0  h1    h2   for      . Let R be the region between the polar graphs of

r  h1   and r  h2   for    

93
If f is continuous on R, then
 h2  
 f  x, y  dA   f  r cos , r sin   r dr d
 h1 
R

If f is non negative on R, then volume V of the region between the graph of f and R is given by
 h2  
V   f  r cos , r sin   r dr d
 h1 

and the area A of R is given by

 h2  
A  r dr d
 h1 

We give the following example to specifically show on how to find limits of integration in polar
coordinates.

Example 4.4.2

Find the limits of integration for integrating a function f  r ,   over the region R that lies
insider the cardioid r  1  cos  and outside the circle r = 1.

Solution

We graph the cardioid and circle and carry out the following steps:
y



2

x
 1 2

Leaves at r = 1 + cos 

 Enters at r = 1
2
Fig 4.4.2: Shaded region represents region between the circle r = 1 and the cardioid
r  1  cos  .

94
Step 1

Holding  fixed, let r increase to trace a ray out from the origin.

Step 2

Integrate from r-value where the ray enters R to the r-value where the ray leaves R.

Step 3

Choose  -limits to include all the rays from the origin that intersect R.

The result is the integral



r 1 cos 
2 r 1 f  r ,  r dr d
2

Example 4.4.3

Find the area enclosed by the lemniscate r 2  4 cos 2

Solution Leaves where r  4cos 2



4

Enters where r=0 


4
Fig. 4.4.3: To integrate over shaded region bounded by the lemniscate r 2  4 cos 2 , we run r

from 0 to 4 cos 2 and  from 0 to .
4

We graph the lemniscate to determine the limits of integration and see that the total area is four
times the first quadrant portion.

95

4 cos 2
Area = 4 4  r dr d
0 0
 r  4 cos 2
 r2 
 4 4   d
0
 2  r 0

 4
 4 4 2cos 2 d   4sin 2  0  4
0
Example 4.4.4

Suppose R is the region bounded by the circles r  1and r  2 and the lines    and    ,
where 0      2 . Express

   3x  8 y 
2 dA
R
as an iterated integral in polar coordinates. Then evaluate the iterated integral for

(a)   0and    R isa quarter - ring 
2
(b)   0and     R isa half - ring 
(c)   0 and   2  R is a ring 

Solution


R
 3 x  8 y   1  
2 dA   2 3r cos  8r 2 sin 2  r dr d

where we have made the substitutions

x  r cos  , y  r cos  and dA r dr d .

(a) If R is the quarter-ring, see figure 4.4.4 below, then


y

r =2

r =2
 0

x
0 Fig 4.4.4

96
 2
      
2
3x  8 y dA  2 3r cos   8r 2 sin 2  r dr d
 1
R
 2

  2  3r 2 cos  8r 3 sin 2  dr d
 1 

 1
2
  2 r 3 cos   2r 4 sin 2  d


  2  7 cos   30sin 2   d
0

  2  7 cos   15  15 cos 2  d
0

 15  2
  7sin   15  sin 2 
 2 0
15
 7 
2
(b) For half ring, only the outer limit  of integration is replaced by  , so we obtain
2
y

r =2
R

 

x
0 r=1
Fig 4.4.5


   3x  8 y  3r cos  8r 2 sin2   r dr d
2
 0 1
2 dA 
R

 15  2
  7 sin   15  sin 2 
 2 0
 15 

97
(c) For the ring we substitute 2  for  in the outer limits of integration obtaining
y

r =2
R
R

r=1  0

x
0

  2

Fig 4.4.6


R
 3 x  8 y  0 0  
2 dA  2 1 3r cos   8r 2 sin 2  r dr d

2
 15 
  7 sin   15  sin 2 
 2  0
 30 

Example 4.4.5

Let D be the solid region bounded above by the paraboloid z  4  x2  y 2 and below by the xy
plane. Find the volume V of D.

Solution

The region R over which the integral is to be taken is bounded by the circle x2  y 2  4, whose
equation in polar coordinates is r = 2, therefore D can be described as the region between the
paraboloid and the disk x2  y 2  4.

98
As a result
V
R
  4  x  0 0  
2  y 2 dA  2 2 4  r 2 r dr d

    4r  r 3  dr d
2 2
0 0
2
2  r4 
  2r 2   d
0  4 
 0
2
 4d  8
0

99
Self Test 4.4.6

In 1- 7 evaluate the double integral by changing to polar coordinates

a a2  y 2 3 9 x2  y
1. 0 0 y dx dy 2. 00 arc tan   dy dx
x

2 2 x  x2 4 4 y  y2
3. 0 0 xy dy dx 4. 0 0 x 2 dx dy

5.
1
0  y
2 y 2
1 dx dy 6.
1
0 0
1 y 2
 
sin x 2  y 2 dx dy

 x2  y2  dy dx
2
1 x x
7. 0  x x
2

8. Express the given integral in polar coordinates, and then evaluate it

  x 
2  y 2 dA
R
where R is the region bounded by the cardioid r  2 1  sin  
1
2
9. Find the area of the region cut from the first quadrant by the curve r  2  2  sin 2  .

10. Find the area of the region cut from the first quadrant by the cardioid r  1  sin  .

Solutions to Self Test 4.4.6

a3 9 2 2
1. 2. 3. 4. 2
3 16 3

  3
5. 6. 1  cos1 7. 8. 35 9. 
4 4 32

8  3
10.
4

100
4.5 Surface Area

In this subsection we show how to find the upper surface area of the solid.

Definition 4.5.1

If f and its first partial derivatives are continuous on the closed region R in the xy – plane, then
the area of the surface z  f  x, y  over R are given by
Surface area    ds
R
2 2
  1   f x  x, y    f y  x, y  dA
R
Remark

The double integral for surface area has some similarity to the integral for arc length
b
length of x – axis:  a dx
b b 2
Arc length in xy-plane:  a ds   a 1   f   x  dx

Area in xy-plane:    dA
R
2 2
Surface area in space:    ds    1   f x  x, y    f y  x, y  dA
R R
Example 4.5.2

Find the surface area of that portion of the plane z  2  x  y that lies above the circle
x2  y 2  1 in the first quadrant, as shown in figure 4.5.1.
z

2
Plane: z  2  x  y

1 1
y
2
2 2 2
R : x  y 1
x Fig 4.5.1

101
Solution

Since f x  x, y   1 and f y  x, y   1 where f  x, y   2  x  y , then surface area is given


by

2 2
S  1   f x  x, y     f y  x, y   dA
R

  3 dA  3   dA
R R
Now, since the integral on the right is simply 3 times the area of the region R, we have

  3
S  3  area of R   3    
4 4

Example 4.5.3

Find the area of that portion of the surface f  x, y   1  x2  y that lies above the triangular
region with vertices (1,0,0), (0, -1, 0) and (0,1,0).

Solution

Let f  x, y   1  x2  y
f x  x, y   2 x and f y  x, y   1

Then surface area is given by

2 2
S  1   f x  x, y     f y  x, y   dA
R

  1  4 x 2  1 dA    2  4 x 2 dA
R R

102
y
1 y = 1- x

R :0  x  1
x 1  y  1  x

y = x-1
-1

From figure 4.5.2 we see that the bounds for R are

0  x  1 and x 1  y 1  x

Thus, the integral becomes


1 1 x
S  x1 2  4 x 2 dy dx
0
1  1 x 
 2  4 x2   dy dx
0  x1 
1 1 x

0
2  4 x2  y x1 dx

   2 2  4 x 2  2 x 2  4 x 2  dx
1
0 
   2 2  4 x 2  2 x 2  4 x 2  dx
1
0 
1
 3

2  2 
  x 2  4 x  ln  2 x 2  4 x  
 
2  4 x2 2 

   6 
 
 0

 
 6  ln 2  6  6  ln 2 
1
3
2
1.618

103
Remark

Many integrals for surface area can be simplified by making a change of variables to polar
coordinates as seen in the next examples.

Example 4.5.4

Find the surface area of the paraboloid z  1  x2  y 2 that lies above the unit circle, as shown in
figure 4.5.3.
z

2 Paraboloid: z  1  x2  y 2

-1 -1
1 1

1 1

x y

Solution

Let f  x, y   1  x2  y 2. Then

f x  x, y   2 x and f y  x, y   2 y.

2 2
S  1   f x  x, y     f y  x, y   dA
R

  1  4 x 2  4 y 2 dA
R

We convert to polar coordinates by letting x  r cos and y  r sin . Then since the region is
bounded by 0  r  1 and 0    2

104
We have
1
3
S
2 1
0 0 1  4r 2 r dr d  
2 1
0 12

1  4r 2 
2
 
0


2 5 5  1
d 

 5 5 1  ~ 5.33
0 12 6

Example 4.5.5
Find the surface areas S of that portion of the hemisphere f  x, y   25  x 2  y 2 that lies
above the region R bounded by the circle x2  y 2  9 as shown in figure 4.5.4.
z

Hemisphere: f  x, y   25  x 2  y 2

5 5
y
2 2
x R: x  y 9
Fig 4.5.4

Solution

The first partial derivatives of f are

x y
f x  x, y   and f y  x, y  
2
25  x  y 2 25  x 2  y 2

and form the formula for surface area, we have


2 2 5
ds  1   f x  x, y    f y  x, y  dA  dA
25  x 2  y 2

105
Therefore the required surface area is

5
S  dA
R 25  x 2  y 2

Now, we convert to polar coordinates by letting x  r cos and y  r sin . Then, since the
region R is bounded by 0  r  3 and 0   2

We obtain

2 3 2  3
 25  r 2  d
5
S
0  
0
r dr d  5
0  
25  r 2 0
2 2
 5 d  5   0
0
10

106
Self Test 4.5.6

In 1 – 5, find the area of the surface given by z  f  x, y  over the region R.

1. f  x, y   2 x  2 y
R: triangle with vertices (0,0,0), (2, 0,0), (0,2,0)

2. f  x, y   8  2 x  2 y
R  x, y  : x2  y2  4
3. f  x, y   9  x 2
R: square with vertices (0,0,0), (3, 0,0), (0,3,0), (3,3,0)

4. f  x, y   2 y  x 2
R: triangle with vertices (0,0,0), (1, 0,0), (1,1,0)

5. f  x, y   x 2  y 2
R   x, y  : 0  f  x, y   1

Solutions to Self Test 4.5.6

1. 6 2. 12 3.
3
4
6 37  ln  
37  6 

27  5 5
4. 5.  2
12

107
4.6 Triple Integrals

Triple integrals are used to calculate the volumes of irregular three dimensional shapes and the
average values of functions over three dimensional regions.

4.6.1 Properties of Triple Integrals

Triple integrals have the same algebrain properties as double integrals and single integrals. If
F  F  x, y, z  and G  G  x, y, z  are both integrable, then

1.    K  dV  K    FdV (any number K)


D D

2.     F  G  dV     FdV    GdV
D D D

3.     F  G  dV     FdV    GdV
D D D

4.    FdV  0 if F  0 on D
D

5.    FdV     GdV if F  G on R
D D

6. If the domain D of a continuous function F is partitioned by smooth surfaces into a finite


number of non overlapping cells D1 , D2 ,..., Dn then

   FdV     FdV    FdV  ...   FdV


D D D Dn
1 2

Definition 4.6.2

The volume of a closed bounded region D in space is the value of the integral

D     dV .
Volume of D

4.6.3. Evaluation of Triple Integrals

Let f be continuous on a solid region h defined by

a  x  b, h1  x   y  h2  x  , g1  x, y   z  g2  x, y 

108
where h1, h2 , g1 and g2 are continuous functions. Then

b h  x  g  x, y 
 f  x, y, z  dV  
2 2
 f  x, y, z  dz dy dx
a h  x  g  x, y 
Q 1 1

to evaluate a triple integral in the order dz dy dx , we hold both x and y constant for the inner
most integration, and then hold x constant for the second integration.

The z-limits of integration indicate that for every (x,y) in the region R, Z may extend from the
lower surface z = a to the upper surface z = b.

Example 4.6.4

Let R be the rectangular region in the xy-plane bounded by the lines x = 2,


5
x  2, x  , y  0 and y   and let d be the parallelepiped between the graphs of z  0 and
2
z  2 on R (see figure 4.6.1)
z

2
D
y
R 
5
2
Fig 4.6.1
x
Solution
5
 2
   zx sin xy dV  22  0  0 zx sin xy dz dy dx
D
5 2
  z2x 
 2
 0  2 sin xy  dy dx
2
0
5

 2  2 x sin xy dy dx
2 0

109
5

  2  2 cos xy  0 dx
2
5
 2 2
  2 x  sin  x 
  0
 2 5   2 
  5  sin    4  sin 2 
  2    
2
 1

We have already seen that there are two workable orders of integration use to evaluate a double
integration. For triple integrals, there are sometimes as many as six workable orders of
integration. In the next example, all six are workable.

Example 4.6.5

Each of the following integrals gives the volume of the solid prism shown in figure 4.6.2.
z

2
x
Fig 4.6.2

1 1 z 2 1 1 y 2
(a)  0  0  0 dx dy dz (b)  0  0  0 dx dz dy
1 2 1 z 2 1 1 z
(c) 0  0  0 dy dx dz (d) 000 dy dz dx

1 2 1 y 2 1 1 y
(e) 0  0  0 dz dx dy (f) 0  0  0 dz dy dx

110
Example 4.6.6

Evaluate the iterated integral

2 x x y x
0  0  0 e  y  2 z  dz dy dx

Solution

Holding x and y constant, we have

x y
 
2 x x y x 2 x x
   e  y  2 z  dz dy dx  e yz  z 2 
  dy dx
0 0 0 0 0 0


2 x x 2
0 0   
e x  3xy  2 y 2 dy dx

Now holding x constant, we have

x
2 x 2 3xy 2 2 y3 
 
2 x x 2
0 0  2

e x  3xy  2 y dy dx  e  x y 
0
 2
 
3 
 dx
0
19 2 3 x
6 0
 e e dx

 
2
  e x x3  3 x 2  6 x  6 
19
6   0
19  e2 
   1
6  3 

In the next example the given order to integration is not convenient and so we change the order
to simplify the problem.

Example 4.6.7

Evaluate
 
3
 0  x  1 sin y
2 2 2dz dy dx.

111
Solution

   
3
 0  x 2 sin y  z 1 dy dx
2 3
 0  x 1
2 2 sin y 2dz dy dx  2

 
 2
 x 2 2 sin y
2 dy dx.
0

the integral  2sin y 2 dy cannot be evaluate by methods of integration we have known so far, so
we change ther order of integration to dz dy dx.
The solid retion D is given by

 
0 x , x y ,1  z  3
2 2

as shown in figure 4.6.3 and the projection of R in the xy-plane yields the bounds


0 y and 0  x  y
2
z

3 D: 0 x 
2

x y
2
2
   1 z  3
 , ,3 
 2 2 
1

x
Fig 4.6.3

112
Therefore we have

D
y 3
0 0 1  
V     dV  2   sin y 2 dz dx dy

 

   
2 3 2
 2 z sin y 2  dx dy  2 2 sin y 2 dx dy
   
0 0 1 0 0
 

   
y
 2 2 x sin y  dy  2 2 y sin y 2 dy

2

0 0 0

2 
  
  cos y  2   cos  cos 
0 2
=1

in the next example we show how to determine limits of integration.

Example 4.6.8

Set up a triple integration for the volume of each of the following solid regions:

(a) The region in the first octant bounded above by the cylinder z  1  y 2 and lying between the
vertical planes x  y  1 and x  y  3 .

(b) The upper hemisphere given by z  1  x2  y 2

(c) The region bounded below by the paraboloid z  x2  y 2 and above by the sphere
x2  y 2  z 2  6 .

113
Solution

(a) z
z = 1 – y2

0  z  1  y2
1 y  x  3  y
0  y 1
1
2 1
y
3
y x=1-y
x
x=3-y
Fig 4.6.4

From Figure 4.6.4, we see that the solid is bounded by the xy-plane  z  0  and above by the
cylinder z  1  y 2. Therefore, we have

0  z  1  y2

Now, by projectin the region onto the xy-plane, we obtain a parallelogram. Since two sides of a
parallelogram are parallel to the x-axis, we set up the bounds

1  y  x  3  y and 0  y  1

therefore, the volume of the region is given by

1 3 y 1 y2
V     dV    dz dx dy
0 1 y 0
D

(b) For the upper hemisphere given by z  1  x2  y 2 , we have


0  z  1  x2  y 2

In figure 4.6.5 below, we see that

114
z
2 2
Circular base: x  y  1 1

1  y  1
 1  y2  x  1 y2
-1
0  z  1  x2  y 2 -1

1 1

y
x
Fig 4.6.5

2 2
the projection of the hemisphere onto the xy-plane is the circle given by x  y  1 , and we can
use either order dx dy or dy dx . Choosing the first, we have

 1  y 2  x  1  y 2 and  1  y  1

which implies that the volume of the region is given by

2
1 1 y 1  x2  y 2
V     dV    1  y2  0 dz dx dy
1
D

2 2
(c) For a region bounded below by the paraboloid, z  x  y and above by the sphere
x2  y 2  z 2  6 we have

x2  y 2  z  6  x2  y 2

the sphere and the paraboloid intersect when z = 2.

115
z
Sphere: x2  y 2  z 2  6
x2  y 2  z  6  x2  y 2
3
 2  x2  y  2  x2
 2x  2
2

z
1

-2 1 1 2

Paraboloid: z  x2  y 2

Fig 4.6.6

Moreover, we see from figure 4.6.6 that the projection of the solid region onto the xy-
plane is the circle given by x2  y 2  2 . Using the order dy dx , we have

 2  x2  y  2  x2 and  2  x  2

which implies that the volume of the region is given by

2
2 2 x 6  x2  y 2
V     dV    x2  y 2 dz dx dy
 2 2
D 2 x

116
Self Test 4.6.9

Evaluate the triple integral

3 2 1 1 x xy
1. 0 0 0  x  y  z  dx dy dz 2. 0 0 0 x dz dy dx

 
4 1 x  x2 sin z
3. 0 0 0 2 ze dy dx dz 4. 0 0 0
2 2 x 2 sin y dx dy dz

1
13 e
z  ln x  dz dx dy
2
5.  15 1  0 x

Solutions to Self Test 4.6.9

1 15  1 
1. 18 2. 3. 1  
10 2  e

2 14
4. 5.
9 3

117
LECTURE 5
Center of Masses and Moments of Inertia
5.0 Introduction

In this section we consider several applications of integration that are related to mass. A
body’s first moments tell us about balance and about the torque the body exerts about different
axes in gravitational field. We restrict our study to centre of masses and moments of inertia of a
two-dimensional and a three dimensional system.

Objectives

At the end of this lecture you should be able to:

 Define moments and centre of mass for a two and three dimensional objects
 Solve problems using cylindrical and spherical coordinates.
 Change variables in Double and Triple integrals

5.1 Definition of the moments and centre of mass of a two-dimensional system

let   x, y  denote the density of the object. Then the table below gives a summary of formulae
on how to determine moments and centre of mass of a two dimensional system.

Density :   x, y 

Mass : M     x, y  dA

First Moments : M x   y   x, y  dA

M y   x   x, y  dA

Center of Mass :  x , y  where


My Mx
x and y 
M M

118
Moments of Inertia:

About the x – axis : I x   y 2   x, y  dA

I y    x2   x, y  dA
About the y – axis :

About the origin :  


I0    x 2  y 2   x, y  dA

 Ix  I y

About a line L : I L   r 2  x, y    x, y  dA
Where
r  x, y   distance from  x, y  to L.

Radii of Gravitation:

Ix
About the x – axis : Rx 
M

Iy
About the y – axis : Ry 
M

I0
About the origin : R0 
M

Definition 5.1.1

Suppose an object with continuous mass density  occupies a solid region D. Then the objects’
moment M xy about the xy-plane, its moment M xz about the xz-plane, and its moment
M yz about the yz-plane are defined by
M xy     z   x, y, z  dV
D

M xz     y   x, y, z  dV
D

M yz     x   x, y, z  dV
D

119
If the mass M of the object is positive then the centre of gravity  x , y , z  of the object is defined
by

M yz    x   x, y, z  dV
X   D
M    f  x, y, z  dV
D

M
   y   x, y, z  dV
Y  xz  D
M   f  x, y, z  dV
D

   z   x, y, z  dV
Z D
     x, y, z  dV
D

moments of inertia about a line L is given by

IL     r   x, y, z  dV
2
where
D

r  x, y, z   distance from point  x, y, z  to line L.

Radius of gyration about a line L is given by

IL
RL 
M

Example 5.1.2

A thin plate covers the triangular region bounded by the x-axis and the lines x = 1 and
y = 2x in the first quadrant. The plates density at the point (x,y) is   x, y   6 x  6 y  6 . Find
the plate’s mass, first moments, center of mass, moments of inertia, and radii of gyration about
the coordinate axes.

120
Solution

We sketch the triangular plate as shown in the figure below.


2
(1, 2)

y = 2x
x=1

0 1

The plates mass is

1 2x 1 2x
M  f  x, y  dy dx    6 x  6 y  6  dy dx
0 0 0 0
 14
the first moment about the x-axis is

1 2x
Mx   y   x, y  dy dx
0 0


1
0 0
2x
 6xy  6 y2  6 y  dy dx
11

The first moment about the y-axis is

1 2x
My   x   x, y  dy dx
0 0
1 2x
 x  6 x  6 y  6  dy dx
0 0
10
the coordinates of the center of mass are therefore

My 10 5
X  
M 14 7
M 11
Y  x 
M 14

121
The moment of inertia about the x – axis is

1 2x 2
Ix   y   x, y  dy dx
0 0


1
0 0
2x
6xy2  6xy3  6 y2  dy dx
y 2 x
1 3 
   2 xy3  y 4  2 y3  dx
0 2  y 0


1
0  40x4 16x3  dx
1
 8 x5  4 x 4   12
  0

Example 5.1.3

Find the mass of the triangular lamina with vertices (0, 0), (0, 3) and (2, 3), given that the density
at (x, y) is   x, y   2 x  y.

Solution

We sketch the triangular lamina


y

y=3
(0, 3) (2, 3)

R
2
x y
3

3x
the region R has the boundaries x  0, y  3 and y  . The mass M of the lamina is
2
2y
3
M    2 x  y  dA  0  0
3  2 x  y  dx dy
R

122
27
3 10 3
   x 2  xy  3 dy   y 2 dy
0 0 9 0
3
 
10  y3 
  10
9  3 
  0

Example 5.1.4

2
Find the center of mass of the lamina corresponding to the parabolic region 0  y  4  x ,
where the density at the point (x, y) is proportional to the distance between (x, y) and the x-axis
as shown in the figure below.
y
Variable density:
  x, y   ky
4

y  4  x2

(x ,y)

2 0 2

Solution

Since the lamina is symmetric with respect to the y-axis and   x, y   ky , the centre of mass
will lie on the y-axis thus x  0.

Mass of the lamina,


k 2  2  4 x
4 x2
2
2
M
2 0   ky dy dx  y
2 2   0  dx


k 2
2  2
 
16  8 x 2  x 4 dx
2
k 8 x3 x5 
 16 x   
2  3 5 
2

123
 64 32  256 k
 k  32    
 3 5  15

Next, we find the moment about the x-axis,

4 x2
2 2 4 x2 2
Mx   y  ky  dy dx  k
  
y dy dx
2 0 2 0
4 x 2
2  y 3 
 k   dx
2
 3  0


k 2
3   2 
64  48 x 2  12 x 4  x6 dx 
2
k 3 12 x5 x7 
  64 x  16 x   
3  5 7 
2
4096 k

105

Thus,

M x 4096k 105 16
y  
M 256k 15 7

 16 
and the center of mass  0 ,  .
 7

Furthermore, the moment of inertia abut the x-axis of the lamina is given by

2 4 x2 2
Ix   y  ky  dy dx

2 0
4 x2
4 x2 3
2 2  y4 
k  
2 0
y dy dx  k  
2 4
  0
dx 

k 2
4  2

256  256 x 2  96 x 4  16 x6  x8 dx 
2
k 256 x3 96 x5 16 x7 x9 
  256     
4  3 5 7 9 
2
32.768k
 .
315

124
and the radius of gyration about the x-axis is given by

Ix 32768k 315
Rx  
M 256 k 15

128
 ~ 2.47
21

Example 5.1.5

Find the radius of gyration about the y-axis for the lamina corresponding to the region

R :0  y  sin x, 0  x  

where the density at (x, y) is given by   x, y   x.

Solution

We sketch the region R as shown in the figure below:


y
R :0  y  
0  y  sin 
2

 
x
2
The mass, M is given by

 sin x  y sin x
M 
0  0
x dy dx 
0 xy  y0 dx
 
 x sin x dx    x cos x  sin x  0  
0

125
the moment of inertia about the y-axis is

 sin x 3  3
Iy   0 x  y  0 dx
0
sin x
x dy dx 
0



  x3 sin x dx   3x 2  6
0   sin x    x3  6x  cos x 0
  3  6

Thus, the radius of gyration about the y-axis is

 3  6
Iy
Ry    2 6
M 

1.97

in the next examples we show how to find centre of mass of solid regions.

Example 5.1.6

Suppose an object occupying the solid region D bounded by the circular cylinder shown in figure
below has a mass density  given by
  x, y, z   20  z 2
z

0 2 y
2

x
Compute the moments M x y , M x z and M y z and then determine the center of gravity of the
object.

126
Solution

The moment about the xz – plane is


M x z     y   x, y, z  dV    y 20  z 2 dV
D D

 2  2  0  
2 2 4
 y 20  z 2 dz dy dx

4
2 2  z3 
  
 2 2
y  20 z   dy dx
 3 
0
2 2  64 y 
  
2 2 
80 y 
3 
 dy dx
2
2  32 2  2
 

2 
40 y 2 
3
y 
 2
dx  
2
0 dx

=0

OR since the axis of symmetry lies on the z-axis it follows that

M x z  0.

the moment about the yz-plane is


M y z     x   x, y, z  dV    x 20  z 2 dV
D D


2 4
2
 2 2 0   
y 20  z 2 dz dy dx
4
2 2  z3  2 2  64 
  
2 2
x  20 z   dy dx 
 3   2  2 x 80  3 
dy dx
0
240  64 2 176 2
x  y  2 dx 
2

3  2 3  2
4 x dx

176  2  2
 2x  0.
3   2

OR since the axis of symmetry lies on the z-axis it then follows that

Myz 0

To determine M x y we use cylindrical coordinates system which we introduce in the next


subsection.

127
M x y     z   x, y, z  dV    20 z  z 3 dV
D D
 

0
2 2 4
0 0   
20z  z3 r dz dr d 
4
2 2  z4 
 
 2
r 10 z   dr d .
0 0 
 4 
0
2
2 2 2  r 2 
  96 r dr d  96 
  d
0 0 0
 2  0
2 2
 96 x 2  d  192   0
0
 384 .

The mass M of the cylinder is given by

M       x, y, z  dV 
D
2 2 4
 
0 0 0  
20  z 2 r dz dr d

4
2 2  z3 
  r 20 z   dr d
0 0 
 3 
0
2 2  64  176 2 2
  r 80   dr d    0 r dr d
0 0  3 3 0
2
176 2  r 2  176 2
  
3 0  2 
 d 
3 0
2 d
0
176 2 704
  2  0  
3 3

Myz Mx z Mx y 384
X  0; Y   0, Z  
M M M  704 
 
 3 
18

11
 18 
center of gravity  x , y , z    0, 0,  .
 11 

128
Example 5.1.17

Find the moments of inertia about the x and y axes for the solid region lying between the
hemisphere z  4  x2  y 2 and the xy plane, given that the density at (x, y, z) is proportional to
the distance between (x, y, z) and the xy plane.

Solution

The sketch of the solid region is as shown in the figure below


z
2  x  2

2  4  x2  y  4  x2

0  z  4  x2  y 2

Variable density
  x, y, z   kz

2 2

y
x the density of the region is given by   x, y, z   kz . Considering the symmetry of this problem,
we have that I x  I y , and we need just to compute one moment say I x .

 
I x     y 2  z 2   x, y, z  dV
D


2
2 
4 x 2
4 x2 0 
4 x2  y 2
 y2  z2   kz  dz dy dx
4 x 2  y 2
2  y2  z2 z4 
4 x 2
     dy dx
 2  4 x 2
 2 4 
0
 2
   
2
2 2 4  x2  y 2 
2 4 x 2  y 4  x  y
 k  
 2  4 x 2  2 4
 dy dx
 
 

129
4 x 2 
 

k 2 2
  
4 2  4 x 
2  4  x 2  y 4  dy dx

4 x 2
k 2  y5 
 
2
   4  x2 y  dx
4 2  5 
   4 x 2
5 5

 
2 2
 
k 2 8 4k 2

4   2 5
4  x 2 dx   4  x 2
5 0
dx

4k 2 6

5 0 64cos  d

 256k  5 
    8k
 5  32 
by use of the substitution x  2sin .

Thus,

Ix  I y  8k 
.

130
Self Test 5.1.8

In exercise 1-4 find the mass and center of mass for the lamnia of specified density.

1. R: rectangle with vertices (0,0), (a,0), (0,b), (a,b)

(a)   k (b)   ky

b 
2. R: triangle with vertices (0,0)  , h  ,  b, 0 
2 
(a)   k (b)   ky

3. The square with vertices  0, 0  , 1, 0  , 1,1 ,  0,1 ; density at  x, y  equal to y tan 1 x.

4. The region bounded by y  x2 and y = x+ 6, situated to the right of the y axis; density at
 x, y  s 2x.
In exercises 5 – 9 find the mass and centre of mass of the lamina bounded by the graphs of the
given equations and the specified density.

5. y  a 2  x2 , y = 0

(a)   k (b)   k  a  y  y

6. y  x , y  0, x  4 ;   kxy

7. y  e x , y  0, x  0 , x  2;   ky

1
8. y  , y  0, x  1, x  1;   k
1  x2

9. x  16  y 2 , x  0;   kx

in exercises 10-13, verify the given moments of inertia and find Rx and Ry . Assume each lamina
has a density of   1.

131
10. Rectangle 11. Circle
y

r
x

x
1
I x  bh3
3
1 1
I y  b 3h I0   r 4
3 2

12. Quarter circle 13. Ellipse


y
y

a
x
r
x
1 1
I0   r 4 I0   a
8 4

In exercise 14-18, find I x , I y , I0 , Rx and Ry for the lamina bounded by the graphs of the given
equations

2
14. y  0, y  b, x  0 , x  a ;   ky 15. y  4  x , y  0, x  0,   kx

16. y  x , y  0, x  4,   kxy 17. y  x2 , y 2  x,   kx

18. Find the moment of inertia of the lamina bounded by the graphs of equation
x2  y 2  b2 about the line x  a  a  b  whose density is   k.

19. Find the mass and the center of mass of the solid bounded by the graphs of
x  0, x  b, y  0, z  0, y  b; and whose density is   x, y, z   kxy .

132
In exercises 21-22, find the centroid of the solid region bounded by the graphs of the
given equations. (Assume   x, y, z   1. )

h 2
20. z  x  y2 , z  h
r

2 2 2
21. z  4  x  y , z  0

133
Solutions to Self Test 5.1.8

a b kab2  a 2b 
1 (a) m  kab,  ,  (b) m  , , 
 2 2 2 2 3 

kbh  b h  kh2b  b h 
2. (a) m  , ,  (b) m  , , 
2 2 3 6 2 2

 4 152   57 75 
3.  ,  4.  , 
 5 75   35 14 

k a 2  4a  ka 4  a 15 32  
5. (a) m  ,  0,  (b) m  , 16  3   0,   
2  3  24  5  16 3  

 
4  e6  1 
 
4
32k  8 k 4  e 
6. m  7. m  , 1  e ,  
 
,  3, 
 7  6 2 
3 4  2 e4  1 9  e  e  
 

k   2 8129k  16 
8. m  ,  e,  9. m  , ,0
2  4  15  7 

14. 15.
kab 4
Ix  32k
4 Ix 
kba3 3
Iy  16k
6 Iy 
3
3kab 4  2ka3b 2
I0  I 0  16k
12
2 3
Ix 
3a Ix 
3 3
2 6
Iy 
2b Iy 
2 3

134
16. 17.
3k
Ix 
I x  16k 56
512k k
Iy  Iy 
5 18
592k 55k
I0  I0 
5 504
4 15 30
Ix  Ix 
5 9
6 70
Iy  Iy 
2 14

k b2 2
18.
4

b  4a 2  19. m
kb5
4
 2b 2b b 
centre:  , , 
 3 3 2

 3h  z  42  x 2  y 2 , z  0
20.  0, 0,  21.
 4   16  x 2  y 2

135
5.2 Cylindrical and spherical coordinates.

So far we have worked with rectangular and polar coordinates. In this subsection we introduce
new space coordinate systems of cylindrical and spherical coordinates.

Definition 5.2.1

In a cylindrical coordinate system, a point in space is represented by an ordered  r,  , z  ,


where

1.  r,   is a polar representation of the projection of P in the xy-plane.

2. z is the directed distance from  r ,   to P.


z
Rectangular coordinates
x  r cos 
y  r sin 
p  x, y , z  zz
 r , , z 

x y
r

x y

To convert from rectangular to cylindrical coordinates (or vicer versa) we follow the conversion
guidelines for polar coordinates.

Cylindrical to rectangular Rectangular to cylindrical


x  r cos , y  r sin  , z  z y
r 2  x 2  y 2 , tan   , z  z
x

Remark

Since the representation of a point in the polar coordinate system is not unique, it follows
that the representation in the cylindrical coordinate system is also not unique.

136
Example 5.2.2

 5 
Express the point  r , , z    4, ,3  in rectangular coordinates.
 6 

Solution

Using the cylindrical to rectangular conversion equations given

5
x  4 cos  2 3
6
5
y  4sin 2
6
z 3

Example 5.2.3

Find equations in cylindrical coordinates for the surfaces whose rectangular equations are
as follows:

(a) x2  y 2  4 z 2 (b) y 2  x

Solution

r 2  4z2 r 2 sin 2   r cos 


(a) (b)
r  2z r 2 sin 2   r cos   

cos 
 
r r sin 2   cos   0. Discard r  0 r
sin 2 
 cos  cot 

r sin 2   cos   0

Next we introduce the spherical coordinate system, where each point is representd by an
ordered triple – the first coordinate is a distance, and the second and third coordinates are
angles.

Definition 5.2.4

In a spherical coordinate system, a point P in space is represented by an ordered triple


  ,  ,   where
1.  is the distance between P and the origin   0

137
2.  is the same angle used in cylindrical coordinates, 0    2

3.  is the angle between the positive z-axis and the line segment op , 0     .

Remark

We pronounce  as rho and  is phi.


z

p  x, y , z 

z  r , , z 


0 y
 r
x Spherical coordinates
y

To convert coordinates from spherical to rectangular and vice versa we use the following
equations.

Spherical to rectangular:

x   sin  cos , y   sin  sin  , z   cos 

Rectangular to spherical:

 
y z
 2  x 2  y 2  z 2 , tan   ,   arc cos  
x  x2  y 2  z 2 
 

the spherical coordinate system is useful when working with spherical shapes or shapes
with center of symmetry.

138
Example 5.2.5

Find an equation in spherical coordinates for the surfaces represented by the following
rectangular coordinate equations.

(a) Cone: x2  y 2  z 2 (b) Sphere: x2  y 2  z 2  4 z

Solution

(a)
x2  y 2  z 2

  sin  cos      sin  sin      cos  


2 2 2

 2 sin 2  cos 2    2 sin 2  sin 2    2 cos2 

 
 2 sin 2  cos 2   sin 2    2 cos 2 

 2 sin 2    2 cos 2 
sin 2 
1 with   0
cos 2 
tan 2   1

 3 
thus   or   . The equation   represents the upper half cone, and the
4 4 4
3
equation   represents the lower half-cone.
4

(b) Since  2  x2  y 2  z 2 and z   cos , the given equation has the following spherical
form.

 2  4  cos   0
    4 cos    0
Discard the possibility that   0,

So
  4 cos   0
  4 cos 

139
5.3 Triple Integrals in Cylindrical and Spherical Coordinates

When working with a solid like a cone or a cylindrical shell that has an axis of symmetry
always take the axis to be the z-axis in a cylindrical coordinate system. Likewise when
working with a shape like a ball or spherical shell that has a centre of symmetry, always
choose the centre to be the origin in a spherical coordinate system.

Cylindrical coordinates can always be used for describing cylinders whose planes that either
contain the z-axis or lie perpendicular to the z-axis. The volume element for subdividing a
region in space with cylindrical coordinates is

dv  dz r dr d .

5.3.1 How to Integrate in Cylindrical Coordinates

To integrate a continuous function f  r , , z  over a region given by inequalities

g1  r ,    z  g 2  r ,  
h1    r  h2  
1     2 ,

evaluate the iterated integral

 2 r h2   z g2 r,  f  r, , z  dz r dr d


 1 r h  z g  r, 
1  1

Integrate first with respect to z. Multiply by r and integrate with respect to r. Then integrate
with respect to  .

Example 5.3.2

Find the centroid of the region enclosed by the cylinder x2  y 2  4, bounded above by the
paraboloid z  x2  y 2 , and bounded below by the xy-plane.

140
z
Solution TOP:
Cartesian: z  x2  y 2
Cylindrical: z  r 2

CM .

SIDE:
Cartesian: x2  y 2  4
Cylindrical: r  2

x BOTTOM: Y
z =0
The sketch of the region is as shown in figure above. The cylindrical coordinate inequalities for
the bounding surfaces are:

g1  r ,    z  g 2  r ,   : 0  z  r 2
h1    r  h2   :0  r  2
1     2 :0    2

The centroid lies on the axis of symmetry, in this case the z-axis, so

x  y  0.
2 2 r 2
Mass, M     dz r dr d
0 0 0
2 2 r 2 2
  
0 0
 z 0 
0 4 d

 8

The moment M xy about the xy-plane is

141
2 2 r 2
M xy     z dz r dr d
0 0 0
2 2  z 2  2 2 r 5
    r dr d    dr d
0 0 2  0 0 2
 
2
2 2  r 6  2 16

0  
0  12  d  
0 3
 0
16 2 32
  0   .
3 3

Therefore,

 32 
  4
M xy

3 
z  ;
M 8 3

 4
and the centroid is the point  0, 0,  .
 3

5.3.3. How to Integrate in Spherical Coordinates

To integrate a continuous function f   ,  ,   over a region given by inequalities

g1  ,     g 2  , 
h1      h2  
1     2 ,

evaluate the iterated integral

 2  h2    g2 r ,  


 1  h1    g1 ,  f   , ,   sin  d  d d
2

To do so multiply f by  2 and integrate with respect to  . Multipy the result by sin  and
integrate with respect to  . Then integrate with respect to  .

Example 5.3.4

Find the volume of the upper region cut from the solid sphere   1 by the cone   .
3
Further, find the moment of inertia about the z-axis.

142
Solution
z



3

 1

x y
Sketch the cone and sphere as shown in figure above. The spherical coordinate
inequalities for the required region occupied by the solid are:

g1  ,      g 2  ,   : 0    1

h1      h2   :0   
3
1     2 :0    2

The volume is the integral of f   ,  ,    1 over the region:



2 3 1 2
V
0   
0 0
 sin  d  r d d
 1
2 3   3 
  
0 0  3 
sin  d d
 0
 
2 3 1 2  1 3
  
0 0 3
sin  d d 
0  3 
 cos   d
0
2  1  1 2 2 
 d    0  
0  6  6 6 3

Further, we can find the moment of inertia of the given solid about the z-axis by the
following procedure:

143
In rectangular coordinates, the moment of inertia about the z-axis, denoted I z is given by

 
I z     x 2  y 2 dv
D

In spherical coordinates,

x 2  y 2    sin  cos      sin  sin  


2 2

  2 sin 2  cos 2    2 sin 2  sin 2 


  2 sin 2  .

Hence
 
I z      2 sin 2   2 sin  d  d d

2 3 1 4 3
  
0 0 0
 sin  d  d d
 1
2 3   5 
  
0 0  5 
sin 3  d d
  0


1 2 3
5  
0 0  
1  cos 2  sin  d d


Since sin3   sin 2  sin   1  cos2  sin  

1 2

   3 sin   cos 2  sin  d d
5 0 0


1 2  cos3   3
    cos    d
5 0  3 
0
1 2  1 1 1 1 2 5
    1   d   d
5 0  2 24 3  5 0 24
2
1 5  1  10  
     
5  24  0 5  24  12

144
5.4 Change of Variables in Double and Triple Integrals

Suppose that a region G in the UV-plane is translated one to one into the region R in the
xy-plane by differentiable equations of the form

x  g  u, v  , y  h  u, v  . y

u
x  g  u, v 
R
y  h  u, v 

G v
0

v
0
Cartesian uv – plane Cartesian xy - plane

A function f  x, y  on R is a function f  g  u, v   , h  u, v   on G. If g, h and f have


continuous partial derivatives and f  u, v  is zero, only at isolated points or not zero then

  f  x, y  dx dy    f  g u, v  , h u, v  J u, v  du dv


R G

The factor J  u, v  is given as

x x
u v   x, y 
J  u, v   
y y   u, v 
u v

and is called Jacobian of the coordinate transformation

x  g  u, v  , y  h  u, v  .

Example 5.4.1 Finding the Jacobian for Rectangular to Polar Coordinates

Find the Jacobian for the change of variables defined by


x  r cos and y  r sin 

145
Solution

By definition of a Jacobian, we obtain

x x
r  cos   1sin 
J  r ,   
y y sin  r cos 
r 
 r cos 2   r sin 2   r

Hence

  f  x, y  dx dy    f r cos , r sin   r dr d
R G
   f  r cos , r sin   r dr d  if r  0 
G

Example 5.4.2

y
4 2 1 2 x  y
Evaluate  
0 y 2
dx dy
2

By applying the transformation

2x  y y
u ,v
2 2

and integrating over an appropriate region in the uv-plane.

Solution

The coordinate inequalities for the boundary of R are

y y
0  y  4 and  x  1
2 2

which simplify to:

y  0, y  4, y  2 x and y  2x  2.

We sketch the region R as shown below.

146
y = 2x

y=4
4

y = 2x-2
3

0 1

To find boundaries of G we solve for x and y in terms of u and v to obtain

x  u  v and y  2v.

The sketch of the region G is shown below


u

v=2
2

u=1
u=0 G

v
v=0 1

We make the following table which shows xy equations for R, uv equations for G and
simplified uv-equations.

147
xy-equations for the Corresponding uv-equations Simplified uv-equations.
boundary of R for the boundary of G

y 2v u0
x uv  v
2 2
y  2v  u 1
x  1 u  v    1  v 1
2  2 
y0 2v  0 v0

y4 2v  4 v2

The Jacobian of the transformation is

x x
r v 1 1
J  u, v    2
y y 1 1
u v .

Hence

y
1 v 2 u 1
2 2 x  y dx dy 
4
   u J  u , v  du dv

0 y 2 v  0 u 0
2
1
2 1 2  u2 
  
0 0
2u du dv 2   dv 
0 2
  0
2
dv   v 0  2  0
2

0
Suppose that a region G in uvw-space is transformed one to one into the region D in xyz
space by differentiable equations of the form

x  g  u, v, w , y  h u, v, w , z  k u, v, w  .
then any function f  x, y, z  defined on D can be defined as

F  g  u, v, w , h u, v, w , k u, v, w   H u, v, w  on G. Furthermore.

   F  x, y, z  dx dy dz     H u, v, w J u, v, w du dv dw
D G

148
where
x x x
u v w
y y y   x, y, z 
J  u , v, w   
u v w   u , v, w 
z z z
u v w

Example 5.4.3

For cylindrical coordinates, the transformation from Cartesian rθz  space to Cartesian
xyz  space is given by the equations

x  r cos , y  r sin  , z  z

The Jacobian of the transformation is

x x x
r  z
cos   r sin  0
y y y
J  r , , z    sin  r cos  0
r  z
0 0 1
z z z
r  z
 r cos 2   r sin 2   r.

Hence
   F  x, y, z  dx dy dz     H  r, , z  r dr d dz
D C

Example 5.4.4

For spherical coordinates the transformation from Cartesian xyz-space is given by the
Equations

x   sin  cos , y   sin  sin  , z   cos .

The Jacobian of the transformation is

149
x x x
  
y y y
J   ,  ,  
  
z z z
  
sin  cos 
 cos  cos    sin  sin 
 sin  sin   cos  sin   sin  cos 
cos    sin  0
 sin  cos    cos  sin  X 0    sin  sin     cos  cos 0   sin  cos2  
 

    sin  sin     sin 2  sin    cos2  sin  
  2 sin 

Hence

   f  x, y, z  dx dy dz     H   , ,   2 sin  d  d d .
D G

Example 5.4.5

Evaluate
y
3 4 2 1  2 x  y z 
  
0 0 y  2
  dx dy dz
3
2
by applying the transformation

2x  y y z
u ,v  , w 
2 2 3
and integrating over an appropriate region in the uvw-space.

Solution

The boundaries of the region D in the xyz-space are:

y y
0  z  3, 0  y  4 and  x  1
2 2
which simplify to

z  0, z  3, y  0, y  4, y  2 x and y  2 x  2

150
We solve the given transformation equations in terms of u, v and w to find:

x  u  v, y  2v and z  3w

xyz-equations for the Corresponding uvw-equations Simplified uvw-equations.


boundary of D for the boundary of G

y 2v u0
x uv  v
2 2
y  2v  u 1
x  1 u  v    1  v 1
2  2 
y0 2v  0 v0

y4 2v  4 v2

z0 3w  0 w0

z 3 3w  3 w 1

The Jacobian of the transformation is

x x x
u v w
1 1 0
y y y
J  u , v, w   0 2 0
u v w
0 0 3
z z z
u v w
6

Hence
y
3 4 2 1  2 x  y z  1 2 1
  
0 0 y  2   dx dy dz   u  w J  u, v, w du dv dw
  
3 0 0 0
2

151
1 2 1

0 0 0
 u  w  6 du dv dw
 
1
1 2  u2 
6  
0 0 2
 uw  dv dw

  0
1 2 1 
 6   2 w  dv dw
0 0  2 
1
1 1  2 1 
 6   2 w   v 0 dw 6 2   w  dw
 
0 2  0 2 
1
 w w2   1 1 
 126     126    0  0 
 2 2 
0 2 2 
 12

152
Self Test 5.4.6

In exercise 1 – 3 , convert the given point from rectangle to cylindrical coordinates

1. (0 , 5, 1, ) 2. (1, 3 , 4) 3. (2, -2, -4)

In exercise 4 – 6 convert the given point from cylindrical to rectangular coordinates

 7
4. (5, 0, 2) 5. (2, , 2) 6. (4, , 3)
3 6

In exercise 7 – 9 convert the given point from rectangular to spherical coordinates

7. (4, 0, 0) 8. (-2, 2 3 , 4) 9. ( 3 , 1, 2 3 )

In exercise 10 – 12 convert the given point from spherical to rectangular coordinates

    3
10. (4, , ) 11. (12, ,0) 12. (5 , ,
6 4 4 4 4

In exercise 13 – 15 convert the given point from cylindrical to spherical coordinates

 
13. (4, ,0) 14. (4, ,6) 15. (2,  , 5 )
4 6

In exercise 16 – 18 convert the given point from spherical to cylindrical coordinates

    7 
16. (10, , ) 17. (6 , , ) 18. (8, , )
0 2 6 3 6 6

In 19 – 21 find the Jacobian of the transformation

u v
19. x  3u  4v, y   20. x  uv, y  u 2  v2
2 6

21. x  ev , y  u ev

In 22 – 23 evaluate the integral by using the given transformation

y x
22.   x  3 y dA , where R is the region bounder by the lines y = 1, y = 4
and x–
R
3y = e; let x = 3u + v, y = u.

153
  xy
2
23. dA , where R is the region bounded by the lines x  y  2, x  y  1, 2 x  3 y  1, and
R
1 1
2 x  3 y  0: Let x   3u  v  , y   v  2u 
5 5

In 24 – 25 evaluate the triple integral

 2cos2  4r 2   2  3 2
24.  2
0 0 0 r sin  dz dr d 25. 0 0 0 e
2  d  d d

Solutions to Self Test 5. 4. 6

  
1. (5, , 1) 2. (2, , 4) 3. ( 2 2 , , -4)
2 3 4

4. (5, 0, 2) 5. (1, 3 , 2) 6. ( 2 3 , -2, 3)

 2   
7. (4, 0 , ) 8. ( 4 2 , , ) 9. (4, , )
2 3 4 6 6

5 5 5 2
10. ( 6 , 2 , 2 2 11. (0, 0, 12) 12. ( , , )
2 2 2

     3 
13. (4, , ) 14.  2 13, , arc cos  
4 2  6  13  

  5  
15. 13,  , arc cos    16. (10, ,0)
 13   6

    7 4  5
17.  3 3 , ,3  18.  4, , 3 19.
 6   6  2

20. 2v2  2u 2 21. e2v 22.


1 2
4

e 3 
2
23.
133
2500
24.
52
45
25.
6
1  e8 

154

You might also like